First Principles

Ok, so I’ve just realized something. There are smart people here who take issue with the fact that CO2 captures/retards/slows down, heat flow. Now of course there are nuances, but at Climate Audit Steve doesn’t like the discussion b/c we’ve all heard it before. I mean that’s what it is, we have all heard the arguments. I can’t even imagine a unique new one.

I’ll give my opinion just to start the discussion, but what will happen is that others will give theirs and this thread won’t have anything to do with me or my crazy views.

My opinion is:

CO2 is more absorbent of heat in longwave than shortwave radiation. Because sunlight absorbed and re-emitted, the energy is converted from short to long wavelength, there is necessarily a warming effect. The questions in my mind are how much, how dangerous and what can we do about it.

My answers are dunno, but less warming than IPCC rubbish, likely beneficial and can’t be changed with today’s technology.

Does that make me a lukewarmer, a denier, a skeptic or what?

191 thoughts on “First Principles

  1. Even a strong negative would result in some warming b/c the feedback is on temperature rather than a chemistry. However, the temp rise doesn’t need to be of a detectable magnitude.

  2. Disagree on feedback needing positive temperature change.

    Boiling water is a good example of pouring energy in but getting zero temperature change.

    Imagine if CO2 catalyses the production of transparium whose presence makes the atmosphere more transparent. You could even get a drop in temperature with more CO2.

  3. Sorry to reply to my own post at #3

    The more likely scenario for negative feedback is the CO2 catalysed conversion of say Opaquium Hydroxide into say Transparium Peroxide.

  4. OK here’s a wild conjecture: Seems to me one of the most important assumptions about the greenhouse effect is that a greenhouse gas absorbs upward longwave radiation and then reradiates exactly half of it back down. Now there are three things that make me vaguely uneasy about that: 1) conservation of momentum – photons carry momentum and the upward radiation momentum vector is somehow being turned 180 degrees in order to radiate back down. If you think about it that photon could be absorbed at the surface, imparting its momentum to the surface, get re-reradiated back up to a GHG molecule, imparting its momentum to the GHG molecule, etc. This would appear to manufacture a momentum explosion out of nothing; 2) In his book “Collective Electrodynamics: Quantum Foundations of Electromagnetism” “Carver Mead has developed an approach he calls Collective Electrodynamics in which electromagnetic effects, including quantized energy transfer, derived from the interactions of the wavefunctions of electrons behaving collectively.[2] In this formulation, the photon is a non-entity, and Planck’s energy–frequency relationship comes from the interactions of electron eigenstates. The approach is related to John Cramer’s transactional interpretation of quantum mechanics, to the Wheeler-Feynman absorber theory of electrodynamics, and to Gilbert N. Lewis’s early description of electromagnetic energy exchange at zero interval in spacetime.” cite: Wikipedia. Mead is no fringe figure. In his approach there are no such things as “free” photons and no electron looses energy except to another electron with which it is in resonance – which implies that no GHG molecule can emit a photon toward the surface unless it can “find” another electron on the surface which is in a suitable electronic state to absorb exactly the amount of energy the GHG molecule has to emit. My conjecture is that if the surface is at a higher temperature than the GHG molecule then the odds are reduced that such a receptive electron will be “found”. That might produce an asymmetric pattern of radiation from the GHGasses in the atmosphere such that they do not emit exactly half their radiation downwards; 3) even if Mead is wrong and GHG molecules can emit photons without a predetermined receptor, if the surface below is saturated with electrons which are already in an excited state because they are at a higher temperature there may be a paucity of electrons physically capable of absorbing the incident GHG emitted photon. Such a photon would arrive at the surface and be reflected, thereby failing to heat the surface and making its way to outer space.

    The thing is: every explanation of the greenhouse effect I have read starts with the idealized black body assumption from classical thermodynamics. Such a black body absorbs all incident radiation at any frequency. But real materials do not because real materials operate quantum mechanically. They are composed of molecules and atoms whose electrons can only absorb photons of frequencies suitable to their electronic configurations. The classical black body model ignores this fact and the introduction of a grey body coefficient does not answer the objection because a simple fractional coefficient just reduces absorption/emission across all frequencies equally. What I am conjecturing is that as the temperature of a body rises it becomes reflective of the longer frequencies. For many people, the idea that heat can flow from colder to hotter appears to violate the second law of thermodynamics. But the retort is usually “This is radiative physics. What, do you think a photon can choose which direction its going to go?” Well, maybe it can.

  5. Unfortunately, my understanding of physics is from my college years many moons ago. However, I did read something interesting while surfing around Dr. Makarieva’s site that sounded relevant to this discussion, and would be interested in getting feedback because their explanation seems quite reasonable.

    “Atmospheric concentration of carbon dioxide does not depend on temperature. Therefore, CO2 accumulation in the atmosphere cannot lead to a runaway greenhouse effect. The absorption band of CO2 covers 19% of the thermal radiation spectrum of Earth. Not taking the absorption band broadening into account, even if the CO2 concentration increases practically infinitely (not twice or thrice, but hundreds of time compared to the preindustrial period), the greenhouse effect will not grow by no more than 20%.”

  6. I think the thing we can all agree on is the world is not going to end because of it. If thats the case then its not really important except government is trying to legislate control of us through it.

    For my dimes worth thinking spatially (Im a designer that’s how we think), gases are simply ‘droplets’ (like clouds/water vapour) if CO2 is only approx. 400 ppm then I cant see it could have any significant effect because of the huge space between droplets. Its like trying to throw a fishing net over a room full of marbles and expect the marbles not to fall through the net.

    I understand the radiation side but as I said – Im visual – dont count it against me 🙂

  7. Further to 5. I found this at Dr. Spenser’s Blog in his post of Jan 12, 2010, “For some unknown reason, it turns out that most of the microwave radiometers’ calibrated Earth-viewing temperatures are slightly influenced by the temperature of the instrument itself…which should not be the case. One possibility is that the exact microwave frequency band which the instrument observes at changes slightly as the instrument warms or cools, which then leads to weighting functions that move up and down in the atmosphere with instrument temperature.”http://www.drroyspencer.com/ IE the frequency at which the instrument absorbs photons of microwave radiation may be a function of temperature of the instrument. If so, this may be evidence of the effect I conjectured above.

  8. So,
    After being inspired by the Hurricane Article,

    Could some of you weisenheimers expend a bit of text on the role of CO2 heat-‘blocking’-blanket-effect in the context of convective cooling?
    What fraction of the heat loss is radiative from the surface vs. radiative from the heat that got carried up high by convective processes?

    I keep thinking that much of the CO2 arguments have an assumption of a ‘crystalline atmosphere’ where convection is not complicating the analysis.
    Is it just me? Just click on my moniker-icon linkie thing for inspiration…
    TIA
    RR

  9. I like to think of it all in terms of hydraulics. There is a constant flow of “heat” or energy escaping the earth, the co2 and other greenhouse gasses create turbulance like putting rocks across a nice smooth flowing stream. The turbulence slows the water down, so there is a build up behind behind (increase in temp) as particles are scattered around before being pushed forwards again by the net flow of energy. Adding more rocks increases the turbulence and increases the depth upstream, but this effect diminishes the more rocks are added downstream. Imagine there are 950 rows of stones forming a dam which water has risen to flow over. The extra co2 is like increasing the rows to 956 rows, which will have a very very small effect. The interesting thing with water when it is flowing and it hits an object, particles are scattered away from the impact like throwing a stone in a mill pond, but when there is a net flow of energy in one direction, the scattered particles colide with those behind with a forwards momentom, if this forwards momentum is greater the particles are not scattered back. This is the difference between sub-critical, critical and super critical flow. When water is flowing fast, throw in a stone and see if the ripple goes backwards as well as forwards. In slow water it will, but in fast water it wont.

    My question, and this is all just hypothetical, is can the particles rising towards space have a forwards momentum that can overcome or reduce the effect of back radiated energy? Could such an effect exist and could it regulate the effectiveness of the “greenhouse effect”?

  10. As a layman I think that there are two fundamental questions to be asked is

    1 “is CO2 a significant greenhouse gas at all ?”

    The world’s warming of less than 0.8°C since 1850 was ~0.5°C before 1940 (when increased man-made CO2 could not have been a factor) and ~0.3°C (or probably less) up until the present.
    The models being promoted by the IPCC ignore the overwhelming 95% effects of atmospheric water vapour and assume that the effects of even low levels of CO2 substantially reinforce the warming effects. This forcing is entirely unproven science.

    So ignoring mythical forcing the contributors are as follows:
    contribution 1850-1940 contribution 1940-2010
    water vapour and clouds ~95% 0.475°C 0.285°C
    all GHGs CO2, methane and others ~5% 0.025°C 0.015°C

    Note that the rate of warming since 1940 has been 75% of the rate of warming from 1850 to 1940.

    Using simple proportional calculations and doubling of the base 1940 level of CO2 from 280 parts / million to 560 parts / million (3.5 times the current additional level) and assuming that all that additional amount were totally man-made, (ie 3.5 times the current additional amount), this could under these calculations only lead to a temperature rise of 0.052°C. Such an increased contributive amount is still so small as to be virtually undetectable and within the margins of error and would have to assume that none of the other factors, such changes as solar radiance, have had any influence whatsoever.

    There is also very clear evidence that because of CO2’s radiation absorption characteristics the influence of increases in CO2 are self-limiting as far as its warming effects are concerned. That is why when CO2 levels were much higher in the past, (more than 5000 parts / million), that runaway global warming did not occur. So even doubling of CO2 levels now could only have marginal warming effects.

    In addition the historic record shows that CO2 level is a lagging indicator of warming (by several 100 years) so a large part of the increasing concentration of CO2 since 1850 will have arisen naturally from warming sea temperatures with consequent release of dissolved CO2 into the atmosphere.

    There is a second question “Is CO2 a pollutant” ?

    Absolutely not: in spite of what is now being taught in schools about it being a pollutant, CO2 is an essential trace gas in the atmosphere: without it photosynthesis, the origin of all life on the planet, does not work.

    The concentration has risen from 280 parts / million (0.028%) since about 1850 to its current level of 360 parts / million (0.036%), an increase of about 28% over the period.
    Photosynthesis stops and plants die at less than 200 parts / million, so the world is still at close to minimum levels of atmospheric CO2 in the atmosphere. CO2 is essential for the growth of all plants, increased concentrations substantially fertilise plant growth and reduce their water demand so that drier environments become more available for cultivation.

    Horticulturalists routinely add CO2 to a level of 1400 parts / million to the air in their greenhouses to enhance plant growth. That is likely to be the reason why plants get sad as soon as you get them home from the nursery. It has been estimated that the current increased CO2 concentration has already increased plant growth worldwide by some 15% + over pre-industrial levels. So any ideas for the sequestration of Man-made CO2, requiring approximately a doubling of the energy input to do so, would be really pointless, because it would only rob the planet of a valuable fertilisation resource at much increased energy costs.

    But now school children are taught that atmospheric CO2 is a terrible pollutant rather than the very stuff of life, that it really is.

    See the drowning puppies in this piece of crude UK government propaganda: http://www.timesonline.co.uk/tol/news/environment/article6867046.ece

    It is as if that the political establishments, (particularly for example the US Environmental Protection Agency EPA, who have just recently declared that CO2 is a dangerous pollutant under the clean air act), and much of the scientific establishment, and Green activists have collectively forgotten their elementary school biology about photosynthesis and the carbon cycle.

    Via plants, Carbon dioxide (CO2) a trace gas in the atmosphere, that all animals (including man) exhale + sunlight + water (H2O) creates and maintains the very Oxygen (O2) we all need to breath and creates carbohydrates, and thus all other organic compounds. That is the real stuff of life. Rile against that and you negate all the world’s biosphere and thus mankind’s ability to exist on the planet earth.

  11. The table in Question 1 did not format well Try again

    ———————————————contribution 1850-1940 ——————contribution 1940-2010
    water vapour and clouds—~95% ———————0.475°C——————————–0.285°C
    all GHGs CO2, methane and others —~5%————–0.025°C——————————–0.015°C

  12. Re: RuhRoh (Feb 4 02:22),

    The Kiehl and Trenberth analysis of the Earth’s energy budget is what’s usually quoted. The annual global average net heat transfer from the surface to the atmosphere by convection is 102 W/m2 consisting of 78 W/m2 of latent heat from evaporation and condensation of water vapor and 24 W/m2 of sensible heat, higher temperature air that rises and warms the cooler air. Net radiative transfer to the atmosphere and space is 66 W/m2. If the atmosphere were highly viscous so convection did not occur, the surface temperature would be a lot warmer, especially in the tropics.

  13. Let me be loose and not too sciency.

    This ref is an old one but not bad.

    Just looking at CO2 as a molecule and not worrying about intermolecular interactions according to quantum statistical mechanics [because NONE of the models are doing that, yet],CO2 isn’t all that great Heat(IR)-wise. It’s main mode of vibration is the symmetric stretch and is IR dead because there is no change in dipole moment. The few IR active modes seen in the reference mean that if CO2 was supposed to be a heat blanket or fence it is a damn poor one, and at LOW concentrations.

    You say you have a great fence. “I built it myself”. You want to show it off to your friends. “Look what I built”. Friends come over, look at the fence and say, “Uh, you built a fence…” You say, “Yes, I did” (proudly). They say, “There’s only three slats in that fence. Don’t you want to have all the slats in before you claim the fence is built.”

    N2O, hmmm, me like better, IR-wise. H2O… WOW! That’s the one!

    meh, we’ve heard it all before.

  14. The issue is not CO2, the issue that matters, the only issue, is Climate Sensitivity to any warming.

    The only question that matters is about feedback. There is no doubt that rising CO2, however caused, will have a warming input, and there is no doubt of the size of it. It is of a magnitude that a rise from 300ppm to 600ppm will, if nothing else changes, cause a rise in global average temperature of 1C. Every subsequent doubling of ppm will deliver the same warming input.

    It does not matter how this rise in CO2 occurs. It could occur as the result of warming, it could be from volcanos, from human activity, from oceanic outgassing.

    The next question is how the climate system responds to that warming input. It will react in the same way to any warming impulse. The issue is not about how it reacts to warming specifically due to CO2. It is about how it reacts to any warming input, no matter where from or how caused. For instance, changes in planetary albedo affecting solar input, changes in intensity of solar radiation, anything. Even random fluctuations.

    It could be that it amplifies it. It could be, for instance, that it causes rises in water vapor content of the atmosphere which lead to greater warming by the same mechanism as CO2 rises produce a warming impulse. Or it could be that it produces feedbacks of a similar sort to those that occur when we drink a hot cup of coffee, and it produces no net warming. These could be due to clouds and rain.

    The calculation of climate sensitivity is entirely dependent on the size of the feedback parameter. It could be negative or it could be positive. The way we can tell which it is has nothing to do with CO2. We have to do two things.

    1) We have to look at past climate fluctuations and try to figure out what it was that produced them and what effects they had. It would be very important, for instance, to understand exactly what happened during the RWP and MWP. What caused them, how they progressed, what the feedback to whatever initially caused them was, why they stopped. Based on these past fluctuations, my impression is that there must be some limiting mechanism in play. Otherwise how do we account for the MWP being followed by the Little Ice Age?

    Incidentally, we also have to consider sensitivity to cooling. Why did the LIA not simply continue into a new proper Ice Age? What put a stop to the cooling that preceded the MWP. Why do we think that simply lowering the warming input (by lowering CO2 ppm for example, or in some other way) will actually deliver long term cooling, and how much will it cool for a given diminution of warming input? This is a seriously important question if one is contemplating action to lower planetary temperatures, and it is a question independent of the warming question.

    2) The second thing is to look at the effects of one forcing that we know for sure is happening. This is the current rise in CO2. We can measure what it is, we can also in principle measure the changes in temperature. If the temperature does not rise, we know that sensitivity to warming impulses is low, and this is probabably because of feedback. But this assessment is fraught with difficulty, because we have to know that nothing else of a cooling nature is going on at the same time. This is how we get to the aerosol difficulty.

    If you argue that sensitivity is high, then you have to account for the lack of recent warming given the undoubted rise in CO2. One way would be to argue that at the same time as there has been warming from CO2, there has been an almost corresponding cooling from aerosols. Or perhaps, a coincidental and almost corresponding cooling from anomalous behavior of water vapor at high levels of the atmosphere.

    Such debates are quite common in the history of science. It is almost always possible to rescue an hypothesis, like the high sensitivity one, from experimental counter evidence, by modifying ones account of other factors. Medical history is full of this – we could rescue the miasma theory of cholera, by supposing that miasma caused cholera, but did not require a swampy environment for its generation, possibly it was the close air of crowded dwellings. The Black Death/ Bubonic Plague /Rat Flea hypothesis is still being rescued in spite of the countervailing evidence that it moved faster than rats could possibly move, by the hypothesis that it was carried by fleas carried by people. Perhaps in their luggage?

    An instructive current example is the heart disease / cholesterol /statin debate.

    The decisive issue about AGW however is not to do with CO2. It is really to do with climate sensitivity to any warming impulse, from any source. We should stop talking about CO2. We should really focus on the sensitivity to warming impulse issue. This is where you can either refute or confirm the theory once and for all. We also need more work on the cooling sensivity. This is where you can prove that lowering the level of warming input will lower temperatures, and by how much. Or you can prove that it is as nutty as trying to lower body temperature by drinking a glass of cold water. Yes, it stops you sweating, so you feel cooler. And your temperature?

  15. #19

    Climate is not purely temperature driven so you can’t say it’s the climate response solely to warming impulses that matters. Climate is not temperature. Climate is a whole host of physical measures that interact in complex ways. One single dimension view of this mix is the measure of temperature. It may be that if you look at principal components that temperature is a complex function of water availability, chemistry, nuclear radiation, orbital mechanics, solar variability, isotope ratios, particulate matter etc etc.

    That is temperature is an outcome caused by changes in these other parameters. But so are a lot of the other parameters.

    If, for instance, you suddenly increased the amount short-lived isotopes in the atmosphere without any warming, the issue would be how sensitive climate was to this impulse. Example, a nuclear war.

    Ditto, if you suddenly put a large amount of particulate matter in the atmosphere, how does it react? Example, Yellowstone National Park redistributing itself over most of the US.

  16. Carbon’s atmospheric oxide CO2 (plant food) the giver of all life on earth, is already doing most of its warming ability, the first 20 ppmv, given that we are currently at 480 to 490 ppmv adding more will have an unmeasurable positive effect, regardless of wether it doubles trebles or quadruples. And it does not matter if it is man made or natural, it is still the same molecule, 1 atom carbon 2 atoms oxygen, PLANT FOOD.

  17. It makes you SANE for one thing but I would place you
    as a Right Wing Lukewarmer !

    Have fun out there – you are not alone.

  18. First principles include another concept that I don’t hear discussed enough.

    Antarctica has an average mean temperature about minus fifty degrees Celsius.

    Modeling: picture a disk of solid H20 two kilometers thick, 14 million square kilometers in area, and 50 degrees Kelvin away from the phase change to liquid.

    How many joules of energy does it take to
    convert the disk into a puddle? How long does it take for the planet to accumulate that many joules from solar irradience — assuming we caught ALL of it and applied it to our hypothetial ice-disk?

  19. I found this link when I was cleaning out old bookmarks a while back.

    Looks like I’ve grabbed it, had it go mostly over my head then forgot about it.

    I think it might fit in with this post.

  20. Interesting discussion. Some are more or less on teh track that I am.

    However, the source of all of this was correctly identified by (I’m not goign to track individuals’ comments… too much work!) the two very active IR bands for CO2. The second thing is the correct, IMO that the ‘trouble’ is climate sensitivity to the slight warming induced by CO2.

    Here are a few conundrums, for me, however. I have read that if oen looks a tthe emission IR spectrum from the earth (from space) it shows that the two CO2 bands are saturated. This means that all of the outgoing radiation that can be absorbed by CO2 is already being absorbed. Oh, and for those who like analogoies here, this is akin to pullin gdown a window darkening shade in a room. How much more ‘dark’ can you make it by ‘doubling of shades’? I mean how much more ‘dark’ can you make a room buy pulling down more room darkening shades? This reduces the ‘sensitivity of doubling of CO2’ to something much more compl;ex. In addtion to that, the absorbtion is related to teh log of concentration, so most of all IR absorbtion by CO2 has already happened, and it will take an enormous addtional amount to have any effect whatsoever. One point of view I have rread/heard that does not get expanded much upon…. perhaps it is flawed.

    The other is the notion of positive feedback with respect to slight warming….. How is this possible when the earth’s average temperature varies by about 4C EACH YEAR? How can we be worried about one fourth of one annual variance over a century (one C/century)?

    Anyway, my 2 cents.

    Roger

  21. Jeff,
    That makes you a realist…a type of person that has been ignored or excluded from the debate until recently 🙂

  22. Jeff, agree with #27 – you’re a realist and open-minded about this issue, something that until a couple of days ago was very rare.

    About CO2 necessarily causing at least some amount of warming: I don’t think it’s that simple. I think it’ possible to imagine – at least in theory – that elevated CO2 levels may trigger mechanisms that increase the earth’s albedo and that the net effect may be cooling.

    One way in which elevated CO2 levels may contribute to higher albedo is through the production of dimethyl sulfide by plankton:
    http://www.co2science.org/articles/V10/N26/B3.php

    Don’t you just love how complicated the earth is? And especially the oceans?

  23. “Does that make me a lukewarmer, a denier, a skeptic or what?”

    It makes you, or means, that you are an honest person.

    If there were more honest people on the AGW supporting side, actual science would have prevailed and, among other things, we wouldn’t be talking about IPCC reports or Al Gore’s movie.

    To me, it is not whether atmospheric CO2 has an effect, either of cooling or warming, on it’s surroundings – the question is “how much”? Because, whether it is is “no effect at all” or even if it is very minor, it would be overwhelmed by all the other forces acting on the atmosphere. It is only if the effect is strong enough to make a difference that it becomes worthy of serious consideration. As you somewhat point out – there is continuing discussion on whether it has any effect and very little that would support that it is one of the major controlling forces.

  24. It strikes me that all these theories are just guesses, some of them extremely well-informed and logical guesses, but guesses none the less. Likewise the GCMs are guesses, because at some point they include approximations, assumed parameters, and selection of variables and factors to include or reject. Given that they are calibrated against temperature records and proxies, and these records and proxies have been at best brought into question, and at worst completely discredited, the GCMs are left swinging in the wind without a reliable test mechanism. So right now I believe nobody on any side of the debate knows what is really going on.
    I take comfort from a crude analysis that says the actual rise in temperature since the 1970’s is less than that indicated by GISS because of their failure to make intelligent adjustments for UHI; that historical records indicate that the MWP was at least a northern-hemisphere phenomenon and quite likely global, and that sea level rises are minimal. So however much CO2 may be implicated in global temperatures, it is not an imminent crisis therefore environmentalists should back out of the CO2 cul-de-sac and start looking at the real issues, such as deforestation, heavy metal pollution, and non-biodegradable waste. Most of all they should try to engage with the real world and understand that cap-and-trade is a colossal scam and that all the people they normally accuse of funding sites like this are ball-deep in it.

  25. What Michel said.

    It is not about CO2. And especially it is not about changes in the radiative-energy transport properties relative to adding CO2 into a homogeneous mixture of pure gasses. The Earth’s atmosphere is not such a mixture. And, the stuff that makes the Earth’s atmosphere to not be a homogeneous mixture of pure gasses is exactly the stuff that makes calculation of the sensitivity an almost impossible to calculate problem. The real problem will never be calculated from first principles. Empirical data and the parameterizations of that information will always be the critical part of the problem. ( I know I should alway never use ‘never’ and ‘always’, but I just did. )

    The hypotheses involved are; (1) at some time in the past, the Earth’s systems were in radiative-energy-transport equilibrium at the top of the atmosphere, and (2) there is presently an imbalance in that equilibrium state and some kind of average temperature of some parts of the Earth’s systems must increase in order to return to an equilibrium state. Note that the energy transport and storage components of the Earth’s energy budget change at all temporal and spatial scales, so we’re talking about some kind of enormous average that annihilates all temporal and spatial heterogeneities. And the heterogeneities themselves are enormous. Consider the range of temperatures that obtain at any time on the planet and how these undergo enormous daily and seasonal cycles.

    The focus of investigations into climate change should be on the energy balance at the top of the atmosphere; investigations should be directed toward demonstration that the hypotheses are correct. At the present time, the range in the uncertainties of the components of the Earth’s energy budget are larger than the magnitude of the effect said to be the dominant driver for the change in the equilibrium state.

    Additionally, the critically important components of the energy budget, the incoming UV that makes it into the Earth’s systems and the outgoing IR, are not known to great accuracy primarily because of the stuff in the atmosphere that has always been there prior to recent increases in the concentration of CO2; the various phases of water. It is the radiative-transport properties of the phases of water that dominate, for example, the Earth’s albedo. The materials introduced into the atmosphere by human activities other than CO2 have radiative-energy-transport characteristics that make the radiative-energy transport problem practically intractable. And impossibly difficult from first principles.

    It has always been interesting to me that the radiative-energy balance problem is critically dominated by phenomena and processes that are among the least understood in the Earth’s systems. Small changes in the numerical values of these essentially unknown properties and their amounts in the atmosphere are used to ‘tune’ calculations of the past history of the thermal state of the systems. I think most people are much more comfortable whenever the critically dominant phenomena and processes are among the most well understood. Under this condition, we can be more nearly certain that we’re getting the right answer for the right reasons. By the same token, if it has been known for over a century and a half what the critical issues are, why haven’t they been better understood by now. In an engineering situation these phenomena and processes would have been the focus of sufficient investigations so that a firm understanding could be obtained.

    Finally, has it been established that the so-called Global Average Surface Temperature, no matter on what basis such a thing is calculated, has a causal relationship to the physical phenomena and processes that govern radiative-equilibrium of the Earth’s systems?

  26. Hmmm. Deja Vu all over again. If the “atmospheric greenhouse effect” exists, please tell me why it is hotter in dry areas than in humid areas, keeping latitude and altitude constant. If the greenhouse radiation mechanism really makes a difference, it should be much hotter in Miami than Phoenix on an average summer day (and night). It is not.

    And don’t give me the “cold desert night” bullshit.

  27. Dan Hughes “Finally, has it been established that the so-called Global Average Surface Temperature, no matter on what basis such a thing is calculated, has a causal relationship to the physical phenomena and processes that govern radiative-equilibrium of the Earth’s systems?”

    No, it’s nonsense on a stick. The plants in your backyard and the heating system in your house only respond to the local temperature. If the global temperature maps predicted by the GCMs were published the game would have been up a long time ago. The few that I have seen from the IPCC reports have temperatures at the poles that are too high by tens of degrees C. I’d bet anything that these are the best results and the other models are worse, a lot worse.

    Several years ago, one of the more fervent modelers thought that all that was left was to get the model input parameters exactly right. He published a stripped down model that people could run on their PCs and ship the results back to him for analysis. The effort blew up when about the half the time the Pacific Ocean froze at the equator.

    Regarding CO2, it’s saturated at a depth of about 30 m above the earth’s surface. It does heat the atmosphere by capturing IR radiating from the heated surface and transferring it via collisions to the air. However, since it is saturated, adding more only means that the heat transfer occurs closer to the surface. Once you’ve captured all the radiation being generated, which is the case now, adding more CO2 does nothing to the heating rate. Doubling CO2 will mean that the heat transfer occurs in the first 15 m instead of the first 30.

  28. JAE @ 10:45am

    Your observation is very important since it kills the positive feedback via water vapor argument.

  29. Perhaps this paper addresses the question best.

    You can download the paper here. (PDF button on upper right)
    http://arxiv.org/abs/0707.1161v4

    It is rather mathematical — by necessity.

    Falsification Of The Atmospheric CO2 Greenhouse Effects Within The Frame Of Physics
    Authors: Gerhard Gerlich, Ralf D. Tscheuschner
    (Submitted on 8 Jul 2007 (v1), last revised 4 Mar 2009 (this version, v4))

  30. #32 Jay

    It’s cooler in wetter areas due to clouds and vegetation.

    If I have my science correct…water vapor in the form of humidity is a greenhouse gas.
    Water vapor in the form of a cloud is a solar reflector which more then offsets the effect of the humidity.

    IMHO The primary difference between the OMG we are all going to burn up and Ho Hum…nothing to see here move along.

    Warmer oceans will mean more water vapor thru evaporation. If it results in more clouds then nothing to worry about..if it results in more humidity without clouds then there is some cause for concern.

  31. I was a firm believer of the greenhouse effect until I read the “Falsification of…” in comment 35 above. Now, I don’t know what to believe.

    The paper includes comments on a number of published papers pointing out errors. It also includes the following comment:

    It is interesting to observe, that until today the “atmospheric greenhouse effect” does not appear
    in any fundamental work of thermodynamics,
    in any fundamental work of physical kinetics,
    in any fundamental work of radiation theory;
    that the definitions given in the literature beyond straight physics are very different and, partly, contradict to each other.

  32. See I thought the best measure of what will happen in certain current or future situations is to see if its happened before and if so, what happened to the earth then.

    This method of scientific enquiry seems to have been ignored from this debate, but since we do have sufficient historical information sources (proxies etc) to pursue the comparisons, why not compare and contrast…

    The theory: CO2 levels continuing to rise for another 100 years at their current rate of rise or more will bring about a run away warming. So the entire theory is built purely on the fear of what will happen beyond near future levels of rising CO2, so if this senario has been reached naturally and even exceeded, but the world did not witness run away warming of the kind imagined in a positive feedback loop, then there is no reason to believe significantly higher levels of CO2 will do this now.

    2+2 = 4 (in the past)

    Unless the laws of maths (physics) change

    2+2 will still = 4 now and in the future

  33. Michel,

    I have seen some evidence that temperature increase vs. CO2 concentration may be a logarithmic function, not a linear function. If it is indeed logarithmic, every doubling of CO2 would lead to an ever-smaller increase in temperature, not a constant 1C per doubling. Now, if it is indeed a linear function, then a constant amount of warming per doubling could be assumed. I will have to do a bit more research on this to verify….

  34. @35 Beat me to it.

    From the PDF:

    ===
    An experimentum crucis therefore is to build a glass house with panes consisting of NaCl
    or KCl, which are transparent to visible light as well as infrared light. For rock salt (NaCl)
    such an experiment was realized as early as 1909 by Wood.

    2.6 Glass house summary
    It is not the “trapped” infrared radiation, which explains the warming phenomenon in a real
    greenhouse, but it is the suppression of air cooling.
    ===

    I could have followed the math about 40 years ago. All I can say now is that I recognize the symbols.

    The paper addresses almost all of the comments in this thread. One of the sections concludes that that the notion of an average global temperature is nonsense on stilts – heavy math.

  35. Makes u a Lukewarmer,

    Keep is acceptance that GHGs warm the planet. All other things being equal.

    That leaves as open questions;
    1. how much
    2. feedbacks
    3. what to do, if anything

    People left out (sorry) are those that argue: C02 cant warm the planet that is:

    1. concentrationists ( its such a small percentage)
    2. Distractionists ( look at the sun spots, I dont wanna talk about c02)
    3. Confusionists ( look at the historical c02 record)
    4. Perpetualists ( it violates the 2nd law)

    Almost everybody can be a Luke warmer: all over things being equal adding GHGs to the current atmosphere will over time warm the planet.

    The key to calling yourself a Lukewarmer is that you take the “skeptic” or contrarian word off the table.
    And, you can say That you believe in AGW, just not all the tenets.

    Rhetorically it puts you inside their camp. Then they cant kick you out of the big tent.
    like being a blu dog democrat.

    You will be surprised how the reactions change when you start a post with I believe that C02 will warm the planet.

  36. #41, there may be more categories we discover in this thread, as i did:
    5. Coldists (adding CO2 leads to cooling)
    6. Lawyers/Clintonists (it depends on what the meaning of ‘global mean temperature’ is)
    7. G&T’ers – they earned an entire new category

  37. Harry said
    February 4, 2010 at 12:33 pm
    #32 Jay

    It’s cooler in wetter areas due to clouds and vegetation.

    This is true. What bugs me, though, is that radiation is IMMEDIATELY apparent. When the Sun comes from behind a cloud you IMMEDIATELY feel the warmth. When you stand beside a fire, you IMMEDIATELY feel the infrared radiation. Therefore, the infrared radiation from the huge amount of water vapor in the air in Atlanta should be very apparent ALL THE TIME–and it should be much stronger than the IR in Phoenix, if the greenhouse effect is dependant upon IR. The evaporation helps keep the air cool, but it has NO effect on the amount of IR at any given time. Something does not “add up” with the greenhouse gas theory (remember, it is a THEORY, not a demonstratable fact).

  38. Re: Roy Lofquist (Feb 4 12:57),

    That whole section in G&T on actual greenhouses is irrelevant. It’s well known that greenhouses work by preventing loss of heat by convection, not trapping thermal IR. But planets don’t lose heat to space by convection. Energy transfer to and from a body in space is all by radiation.

    The vibrational mode that is most important for CO2 is bending, which does change the dipole moment and results in a very strong band at 667 cm-1, which is near the center of the thermal emission spectrum at 200 to 300 K. If CO2 only absorbed radiation, then the saturation argument would be correct, but the energy of the band is such that it can be excited by collision with other molecules at the temperatures found in the atmosphere. That means CO2 emits as well as absorbs. We know this is true because we can measure the emission spectrum and see the features associated with CO2. The emission rate is a function of temperature. More later. My keyboard is acting up.

  39. Steven Mosher,

    On what basis can you say, ” I believe that C02 will warm the planet.” in contrast to, ” I know CO2 has the potential to warm the planet” ?

    And, what is my label if I say, “Going up or going down, I don’t care so long as we know which way it’s moving”?

  40. @40, Roy:

    I have no reason to doubt the math. I am hoping that someone who fervently believes in AGW can take the paper on and point out the flaws. That would make for an interesting debate. 🙂

    So just bone up on your EM and Radiation theory for when the day comes…

  41. The idea that an increase in atmospheric CO2 content will cause a persistent net increase in the heat content of the planet is what’s called a ‘hypothesis’. You might have all the theoretical reasons in the world for thinking it’s a viable hypothesis, but hypotheses in general are verified only by empirical observations, ideally experiments that use controls to establish causality.

    As such experiments are damned near impossible on a planet wide scale, we have for the most part relied on retrospective analysis of historical observations. These observations cannot establish causality – at the very best they *could* falsify the hypothesis or partially falsify it by placing bounds on the size of the effect (i.e. assuming some relationship between CO2 content and temperature, determine the largest effect the historical data supports).

    Our modelling of the climate system might eventually reach the point where we are confident that experiments conducted on the models are sufficiently accurate to replace empirical observations of the actual Earth system, but we aren’t anywhere near that point yet – and even if we do eventually get there, such results are still not entirely scientific. They might be more compelling than what we have today, but they won’t prove the effect in any meaningful scientific sense.

  42. Harry said
    February 4, 2010 at 12:33 pm
    #32 Jay

    It’s cooler in wetter areas due to clouds and vegetation.

    Harry, thank you! While not a climatologist, I noted that when traveling the Imperial Desert, that the temperature dropped a few degrees when going into the arable area. I noted to my wife and son that it was due to the vegetation, and they poo-poo-ed me! I made the statement under the belief that the sand and soil absorb more heat than living plants.

    Now I get confirmation. Thanks again!

  43. Yeah, I know someone will reply to my previous post: “well, the amount of radiation is dictated by the temperature of the air, so of course you don’t have more radiation in Atlanta than in Phoenix, when the sun is shining.”

    True, but what this says to me is that the IR is the RESULT of heating the air, not the CAUSE of heating the air.

    You can add all the greenhouse gases you wish in a wet area and all you will do is evaporate more water. You will not heat the air more by adding more “greenhouse gases!” When you have evaporated enough water, it will rain, cooling everything off! It NEVER gets very hot (above about 33 C) in the tropics for this reason. CO2 can have NO effect there, except maybe making it rain more (and thereby COOLING the area!).

    Where water is not available (deserts), the air gets much hotter when the Sun is shining, because there is little loss of heat to evaporation.

    All this has nothing to do with radiation from GHGs. It is simple heat storage.

    Besides evaporation, there is the powerful convection effects. The hotter the air gets, the more convection occurs, limiting the amount of heating. This is the primary limiting mechanism of temperatures in the deserts. As admitted by the “warmers,” true greenhouses get hot because of limited convection. What they don’t want to admit, is that the same thing happens in the open atmosphere.

    Yeah, I argue with myself all the time…

  44. Simple models that talk about radiation do not work. Everything changes at the tropopause. In the troposphere heat transport is by convection and extreme amounts of heat are transported in the form of latent heat tied up in the water vapor. Clouds control the radiation transport. At the high density in the troposphere, any absorption of IR by a CO2 molecule will not be re-emitted before before a collision with another molecule. The decay time for re-emitting an absorbed photon is much longer than the mean time between collisions. The energy will be shared equally with all the other molecules. Almost all of the IR emitted from from gasses in the troposphere will be emitted by water vapor as it is much more common. How will this effect clouds? Donno.

    In the stratosphere above the tropopause, temperatures increase with height and there is little convection. Now one can can start to discuss radiation transport as being dominant, but remember that collisional frequency vs. radiation decay time. They do not become equal until at about 50 Km altitude where the density is very low. There is much less water vapor as it is frozen out at -50°C at the tropopause. Now CO2 and ozone and methane become important emitters of IR. O2 and N2 do not absorb or emit IR. Green house gases will start to cool the stratosphere at an altitude where there is not enough IR absorbers above to have a high probability of recapturing the IR photons.

    This why where Miskolczi’s discussion about dynamic equilibrium becomes important. He claims that the atmosphere will maintain a constant mean optical density. Very important data confirms or at least is in agreement with his theory. The humidity at 300 mb and above, where radiation transport becomes important, has dropped over the last 50 years as CO2 has risen.

  45. @47 Willr

    That would be interesting. However the paper states, in the title, that it is written from a physics perspective. I don’t recall that any of the AGW proponents have either a post graduate physics or post graduate mathematics background. The paper is, I presume, an avocation of the authors and not the result of a specific grant or tasking. Who’s going to commission (pay for) a counter analysis?

    @48 Joshv

    “Our modelling of the climate system might eventually reach the point where we are confident that experiments conducted on the models are sufficiently accurate to replace empirical observations”

    I left the study of physics and mathematics about 45 years ago and went into computers. I have followed the literature on computational complexity almost since its inception. The problem of modeling the atmosphere is of type np-complete. Or, in the words of Roger Penrose, non-computable. This means that no conceivable computer operating over any conceivable time period can solve the problem.

    Probably the best explanation of these concepts is in:

    “The Emperor’s New Mind” by Roger Penrose, Oxford University Press, 1989.

    I bought the book when it was first published and it is still on my desk.

    p.s., Penrose has the office next to Stephen Hawking.

  46. In the IPCC report they state that a warmer world is a less arid world, based on paleo evidence (not models!) – hence as the earths mean temperature increases it gets wetter. Lets think about that a minute, why does it get wetter? warmers seas, enhanced hydrological cycle, increased evapotranspiration which leads to increased cloud cover, more convective storms over the tropics etc… these are cooling effects, which most likely occur in response to the external forcings that caused previous warm periods – i.e. the earths climate regime changes to control the external change. Rainfall and clould cover would only need to increase by a couple percent to stop the IPCC’s hypothesised warming. The planet is self regulating, constantly over and undershooting to try and stay around some equilibrium point, hence the climates non-linear chaotic nature.

    Lets look at 50 years of warming, according to the IPCC the increased co2 will increase air temperatures and warm the ocean, the ocean transports this heat to the poles and the warmer sea increases watervapour creating a feedback loop that results in the stable earths climate going out of balance and warming forever. This same thing happens during the day, but much faster. Sun comes up, warms air, warms sea, humidity increase, if the sea gets to warm then it clouds over and things cool or at least dont get any hotter, if its still to warm we get a good thunder storm to further cool things down. On a day to day basis we see the climate sensitivty in action and it works to a) limit warming and b) get rid of excess. This happens every day at the equator and this regulates helps regulates the input to the currents that distribute heat to the southern and northern extremes. If the mean temperature gets a little higher, it will simply cloud over a little earlier at the equator each day and counter the effect, hence no positive feedback! Just a general small increase in mean air temperature for most areas that would only be observable when its cold – and most beneficial!

    Remember, climate is simply expected weather predicted by models or calculated from weather statistics – weather is actual climate

  47. @53 Roy L.

    Yes I work in NP problems and large system modeling so, like you, I am familiar with the idea. Plus I did my time with EM math and theory etc. 🙂

    Mann for example has a physics background. Others have as well. Let them argue with the authors. 🙂

    I looked at the discussion in the other blog — it was interesting — but not many seem to want to take on the math. http://rabett.blogspot.com/2008/02/all-you-never-wanted-to-know-about.html

  48. Will, Peter, #35, #37 and following, I basically agree. My take on the Gerlich and Tscheuschner paper was it was written at the advanced undergrad level, sorta a textbook format using their temperature, entropy, etc analysis as a good, long sample problem in engineering thermodynamics. I couldn’t see a thing wrong with any of the S(U,V,Ni) derivations nor the discussion of basics like intensive vs. extensive physical properties, especially why one should not expected to be able to simply add the effing temperatures and take an average:)

  49. #41, re: Does that make me a lukewarmer, a denier, a skeptic or what?

    lukewarmer is probably the right one but I want to be a lukecooler.

    That could be because I want to call it being a Cool Hand Luker.

  50. After reading some of this blog I decided to brush up on my recollection of IR spectroscopy and also GHG theory. I found this:
    http://www.nov55.com/ntyg.html
    I can’t vouch for how correct it is, but it is very relevant and I would sooner believe it than the alarmist stuff.

  51. #60

    “This means there is no radiation left at the peak frequencies after 10 meters. If then there is a doubling of CO2 in the atmosphere, the distance of absorption reduces to half, or 5m. A reduction in distance is not an increase in temperature. ”

    The reduction in distance is an increase in temp so this statement is wrong. The energy is absorbed and then re-emitted, it doesn’t go away. Shortening the distance increases the time of residence in the atmosphere resulting in some warming. My whole argument is that it may not be very much. Hell if there’s a bit of negative cloud feedback for a slightly warmer temp, the CO2 warming can be decreased even from the zero feedback model.

  52. Re: Gary P (Feb 4 14:14),

    At the high density in the troposphere, any absorption of IR by a CO2 molecule will not be re-emitted before before a collision with another molecule. The decay time for re-emitting an absorbed photon is much longer than the mean time between collisions. The energy will be shared equally with all the other molecules. Almost all of the IR emitted from from gasses in the troposphere will be emitted by water vapor as it is much more common

    Not a total failure, you got one thing correct, energy will be shared equally. This is called local thermal equilibrium or LTE for short. You’re not entirely wrong about re-emission either. LTE requires that energy transfer between molecules be mostly by inelastic collision. But that doesn’t mean that there are no molecules at any given time that have sufficient energy to emit radiation. The Boltzmann distribution determines the fraction of molecules that have as much or more than a specific energy and this fraction for the photon energy at 667 cm-1 is on the order of a percent or 3 for atmospheric temperatures from the stratosphere on down.

    The decay time is only relevant for a free molecule. How do we know this, well, one can measure the atmospheric emission spectrum. Currently the device of choice is a Fourier Transform Infrared Spectrometer. If you were correct about CO2 not emitting then there would be no radiation observed from the ground looking up or at 20 km looking down in the region around 667 cm-1. But as you can see from this figure from A First Course in Atmospheric Radiation, 2nd edition, Grant W. Petty, that’s not true. If you really want to understand atmospheric radiation transfer, don’t waste your time on G&T or Miskolczi (at least not until you have a proper understanding of the standard model), get this book. We can calculate atmospheric emission spectra and they are as close as never mind to the observed spectra if we know the atmospheric conditions at the observation point. The two lower graphs in this figure were calculate using conditions approximating those for the observed spectra. Note that the width of the CO2 band centered at 667 cm-1 is a significant fraction of the total spectrum.

    As far as most of the radiation coming from water vapor, if your comment about decay time were true, water vapor wouldn’t radiate either. Using the same program (Archer MODTRAN interface), we can see the relative contributions by water vapor and CO2 by calculating the total emitted energy summed over the spectrum with water vapor and CO2 present, then set them to zero (or near zero in the case of water vapor) and see how much the emitted energy changes.

    For the 1976 standard atmosphere conditions with a surface temperature of 15 C or 288.2 K, the emitted radiation for 375 ppmv CO2 and water vapor scale of 1 is 258.799 W/m2 at 100 km looking down. Set CO2 to zero keeping everything else constant and the energy goes up to 286.242 W/m2 or a difference of 27.443 W/m2. Set the water vapor scale to 0.0000001 and the emitted energy goes up to 336.608 for a difference of 50.366 W/m2. IR emission from the surface is 360.158 W/m2. There’s still some water vapor left above 10 km, so the total difference in the complete absence of water vapor would be a little higher, but no more than ~10 W/m2, so the contribution of CO2 is about 1/2 that of water vapor, not 1/20th. Methane, ozone, nitrous oxide and a host of other greenhouse gases are still present so the emission to space is less than the emission from the surface. Setting methane and ozone to zero increases the emission to space to 346.97 W/m2 for another 10.36 W/m2. The remaining absorption is due mainly to nitrous oxide and water vapor.

    The fact, and this is by observation not speculation, that the radiant energy emitted to space at the top of the atmosphere is less than that emitted by the surface is the (mis-named) greenhouse effect.

  53. Makes u a Lukewarmer,
    Me too…

    YOU KNOW THAT one piece always missed is that CO2 also reflects the ir coming into the planet! it is not just a one way valve.

    acceptance that GHGs warm the planet. All other things being equal.

    Almost everybody can be a Luke warmer, adding GHGs to the current atmosphere will over time warm the planet.

  54. “Hell if there’s a bit of negative cloud feedback for a slightly warmer temp, the CO2 warming can be decreased even from the zero feedback model.”

    Doesn’t sound like a stable system at equilibrium. Esp if you think temperature should behave something like T = T0 +dT/(1-f) where the dT is due to added CO2 and f is feedback (positive or negative). In that case, it will be at least as much as T0.

  55. It is refreshing to read your positions and actually question where you stand on the warming issue. The test of a true scientist is that he will go where the data takes him, regardless of his personal position. Thomas Huxley considered himself an agnostic about science, which seems a good position to take, if that can be considered a position. My jr. high science teacher used to say “the more you know, the more you know you don’t know.” In my engineering work, that was similar to what we called “analysis paralysis.” That is a description of how the analytical types were never able to finish their analysis because after each analysis they found different conditions that they should consider.

    I am aware of the ability of CO2 to absorb outgoing longwave radiation (not that I understand it) I agree with your questions on the impact on climate. What I have frequently searched for and not found is literature on measurement of the radiation from greenhouse gasses back to earth. They have instruments that can scan a child’s forehead or ear to take his temperature and instruments to scan your house to see your heat loss. This is gross radiation measurement, but I would think one could make instruments to specifically pick up radiation from the C02, H20, CH4 and other bands. In fact, if you read Jim Hansen’s biography on Wikipedia, you see that early on he studied the infrared radiation from Venus. I would think that he would back up his AGW theories with empirical data. There have been satellite measurements of earth’s infrared radiation that shows a reduction (relative to what?) of infrared radiation in the bands for CO2 & H20. If anyone knows where I can get an inexpensive infrared radiation measuring device, please let me know. I could have a lot of fun with that.

    Your article about the hurricanes was very thought provoking. What is intriguing is the hurricanes function as a heat transfer mechanism to move latent heat of evaporation at the earth’s surface and dump it in the upper atmosphere where a significant portion would be radiated to space. (Has any satellites measured radiation given off by hurricanes?) I read one paper that described how an El Nino acted as a heat pump transferring a significant amount of radiation to space. Apparently the oceans accumulate heat during and La Nina and then dump it during an El Nino. Another thing that I have never seen, not that it doesn’t exist, is heat loss from lightning. Most lightning is from cloud to cloud, so at that altitude also, a significant portion of the infrared radiation would go to space. More things to muse about.

  56. One thing I have never seen mentioned in any AGW discussion is the internal temperature of the earth. A quick glance at Google yields a factoid that the temperature rises 3 degrees F per mile of depth. Wouldn’t internal heating enter into the equations somewhere? They all seem fixated on direct solar radiation and re-radiation from the surface.

  57. I think there are 2 things that determine the response (mainly): 1) water vapor and 2) clouds. The IPCC models assume adiabatic changes with altitude which determines moisture content. In still air, fine. But the atmosphere is a boiling pot where hot air rises, and the downward falling air is very dry. If you get more convection in a warmer world, it is possible to dry the atmosphere on average because you speed up the hydrologic cycle. Think of thunderstorms in the summer. A drier atmosphere will let more heat escape (as in the desert). The effect of clouds is related to the above. If the hydrologic cycle is faster, the type of clouds that trap heat are shorter-lived. This basic idea is stolen from Spencer and is related to Willis’ post.

  58. Re; DEwitt 63;

    OK, I followed your note. I have a question.
    What happens to the CO2 molecules after they absorb the radiation?

    Can they absorb multiple photons and get really excited?

    Where does that LWIR energy go after it is absorbed by the CO2?

    I’m missing part of the picture, probably it is just obvious but maybe you could express this part explicitly.
    TIA
    RR

  59. You will be surprised how the reactions change when you start a post with I believe that C02 will warm the planet.

    Steve Mosher, I think, no I’ll make that I know, that if that approach were to work it is because you are talking to a disinterested scientist or layperson, for that matter.

    The nastier and emotional reactions, from both sides, arise from which side you come down on with regards to what we should (could) do about climate warming and the potential beneficial/detrimental effects of the warming that you expect. I do think that people who are inclined to use labels will use them with little regard to these details.

    However, if you want to go down that path I would suggest that on meeting a potential adversary you quickly blurt out that you believe the warming will be x degrees K per century and have y beneficial and z detrimental effects and that the government should do w for mitigation. Then after catching your breath you can tell them what probabilities you put on all of the above.

    And then and only then should you ask them if they (could) love you.

  60. Now let’s talk about heat transfer between infinite parallel planes. The Earth isn’t infinite, but it’s big enough that it’s a reasonable first approximation. Radiative heat transfer from one plane to the other is proportional to the difference between the fourth power of the absolute temperature of each plane corrected for the emissivity. This is basic radiative heat transfer physics and should not be controversial. If it is to you, too bad. I don’t have the patience of Willis Eschenbach so I’m not going to debate that topic. For radiative energy transfer to a surface, radiation can be absorbed, transmitted or reflected only. If we assign a number from zero to one for the fraction of each, they sum to exactly one. For a solid body, Kirchhoff’s Law applies and emissivity at any wavelength is equal to absorptivity. The maximum energy that can be emitted at a given frequency or wavelength is defined by the Planck equation. The actual emission is the Planck value times the emissivity for the frequency or wavelength of interest. The integral of the Planck equation over the entire spectrum assuming constant emissivity is the Stefan-Boltzmann equation. If you don’t know what these are, look them up. Google is your friend.

    So if we are talking about radiating from the Earth’s surface to deep space we have one surface at 288 K (average for the purpose of this calculation) and the other at 2.7 K. Assuming an emissivity of 1 for deep space and 0.98 for the Earth, the rate of transfer is 383.34 W/m2. In fact, we neglect radiation from deep space because it’s insignificant. Then we we also avoid problems with spherical geometry. But we know the average incoming energy from absorbed sunlight over the entire surface is 342 W/m2 maximum (1368/4, spherical geometry with sunlight only on one side of the sphere) and with the relatively high albedo of the Earth it’s much less than that, about 235 W/m2. So how can the temperature of the surface be that high?

    To explain that I have to introduce the concept of the brightness temperature. That’s the temperature of a black body (emissivity/absorptivity exactly 1) that emits the same total radiant power. To convert radiant power to temperature in degrees Kelvin we use the Stefan-Boltzmann equation: P = 0.0000000567*T^4. In order to reduce the net radiant energy transfer from 383.34 to 235 W/m2, there must be something with a brightness temperature much higher than 2.7 K between the surface and deep space. A black body that emits 148.34 W/m2 would do it. For the purposes of this model, we’ll make the shell perfectly transparent to the solar spectrum and perfectly absorptive/emissive in the thermal IR range. We’ll also remove the atmosphere and make the Earth’s surface 68.7%% absorptive and 31.3% reflective for the solar spectrum and and an emissivity/absorptivity of 0.98 for the thermal IR. If we place the shell a few meters above the surface, spherical geometry on the inside can be neglected.

    That would convert to a brightness temperature of 226.16 K for the shell. If we make that a thin plane with one side exposed to deep space and the other to the Earth’s surface, the plane or shell (Willis Eschenbach’s Steel Greenhouse, sort of) would also radiate 148.34 W/m2 to deep space. But that not enough. Incoming solar energy is 235 W/m2 and the shell is only emitting 148.34 W/m2 to deep space. It’s also emitting 148.34 W/m2 towards the surface for a total of 296.68 W/m2, but it’s absorbing 383.34 W/m2 from the surface. Willis fixed that by adding a second shell. I’ll fix it by making the shell partly transparent in the thermal IR. If the transmissivity of the shell is 0.226, the reflectivity is zero, the absorbtivity/emissivity will then be 0.774. Then the shell absorbs 296.7 W/m2, emits 296.7 W/m2 (148.34*2) and transmits 86.6 W/m2 for a total emission to space of 235 W/m2. The brightness temperature of the shell is still 226.16 K, but the actual temperature corrected for emissivity will be 241.1 K.

    The Earth with an atmosphere is more complicated, there’s convective transfer from the surface to the atmosphere and direct absorption of sunlight by the atmosphere, reducing the input energy to the surface from that source, but the principle is still the same. If we use the power numbers from the Kiehl and Trenberth energy balance, the Earth’s surface has a brightness temperature of 288 K. The atmosphere viewed from the surface has a brightness temperature of 274.94 K and the top of the atmosphere has a brightness temperature of 242.2 K, corrected for the 40 W/m2 transmitted directly from the surface, or an overall brightness temperature of 253.7 K.

    How the atmosphere can have a lower brightness temperature from above than below is another subject. But the fact, observation again, that it does is why the analogy of the atmospheric greenhouse effect to insulation is reasonable.

  61. Re: Ruhroh (Feb 4 18:00),

    What happens to the CO2 molecules after absorption of a photon is that 19 times out of 20 or so, the energy is lost to another molecule of the atmosphere by inelastic collision. That’s the fundamental concept behind LTE. But the same applies in the other direction. LTE requires that the energy distribution over all the molecules in the local vicinity follows the Boltzmann distribution. So some small fraction of inelastic collisions with other molecules in the vicinity transfers enough energy to a CO2 molecule that it reaches an energy state high enough to emit a photon from a line in the 667 cm-1 band. At LTE, multiphoton absorption is highly unlikely.

    The mean time between collisions is on the order of 1 nanosecond near the surface, but only a fraction of those collisions are inelastic, so the average lifetime in the excited state is much longer than 1 ns.

  62. 75:

    “The actual emission is the Planck value times the emissivity for the frequency or wavelength of interest.”

    Huh?

  63. Chicken or egg? Are DeWitt’s radiation figures due to the temperatures of the various layers or are the temperatures due to the radiation. I take the first view, that’s all.

  64. #78 The UAH satellite is in a station keeping orbit. This means that corrections to the signal are very limited and the warmth is probably true. We’re in a very strong el nino right now though.

  65. 79. I didn’t mean to imply that the data were incorrect. Rather, I found the warmth surprising and I think this will be bandied about as proof the ‘deniers’ are wrong.

  66. #80, Sorry, other readers will recognize that I’ve done a lot of posts on satellite data quality. It was only recently that UAH switched to the station keeping satellite. Prior to the change the adjustments to measurements were pretty severe.

  67. “63 DeWitt Payne
    Thanks for the book reference. It looks much better than the English translation (poorly done) of a Russian textbook I have been trying to read.

    Sometimes I allow myself to speculate well beyond what I know in hopes of getting more input. I however will stick to my point that there is a distinct change in the mechanism of heat transport at the tropopause. Sure the IR bands of CO2 are absorbed up to this level and CO2 will be re-emitting some IR, but I sure am tired of the naive descriptions that only talk about radiation absorption and and emission and ignore collisions and thermal equilibrium with the other gases.

    Do you have any thoughts about how the extra heat in the troposphere from the extra CO2 absorption is interacting with the heat transport by water vapor and the formation of clouds? I will speculate that it is causing more convection and pushing water vapor higher resulting in a higher and colder tropopause. Perhaps this is what causes the drop in humidity at high altitudes as the water is frozen out at a lower temperature. (The tropopause is higher and colder over the equator than the poles.)

  68. Re: Jeff Id (Feb 4 21:02),

    Recently being about 2002 when Aqua went into service. That’s also the satellite with the best ice detector. It’s the one JAXA Uni-Bremen and Uni-Hamburg use. There must be a data transmission problem as there hasn’t been an update by JAXA since 2/1.

    Btw, the Greasemonkey script killfile works here. There are two people so far whose comments I don’t have to even see much less read or answer.

  69. “Btw, the Greasemonkey script killfile works here. There are two people so far whose comments I don’t have to even see much less read or answer.”

    LOL, one is almost certainly moi. Too bad, cuz he has a mistake in 75. He means hv, not he. He he he.

  70. Re: Gary P (Feb 4 21:30),

    Any warming is going to cause expansion which will raise the height of the tropopause. The tropopause, being a temperature inversion, will act to block vertical convection. But vertical convection becomes less important with altitude even in the troposphere, with the exception of the Intertropical Convergence Zone (ITCZ) and its thunderstorms. Horizontal convection, or advection, as in the Hadley, Ferrel and Polar cells and the jet streams tends to dominate heat transfer at high altitude in the troposphere.

    As far as clouds and rain, your guess is as good as mine. I don’t know enough about it to speculate.

    Another reference you might find interesting is University College Dublin’s Rodrigo Cabellero’s Physical Meteorology lecture notes (~4 MB pdf).

  71. Face the cruel facts, folks. CO2 levels have been as high as 2,000 ppm, IIRC. So, if the GHG THEORY is correct, the Earth would have “melted” many times in the past. Please don’t believe those that continually say that the sky is falling. It has ALWAYS been wrong: ozone hole, DDT, Alar, population bomb, peak oil, etc., etc., etc., ad nauseum!

  72. #85, I certainly did not interpet Planck value in #74 to be the Planck constant nor Planck’s equation to be (E=hv). With just a couple of minutes of googling. What was the reason for stating your purported academic credentials on the previous thread?

  73. #74 It’s astounding that you write “don’t have the patience of Willis Eschenbach” then write an entire book on the topic. A couple of graphs and links and you’ve done a whole post.

  74. @DeWitt Payne #86

    Thank you for the reference. Also for your other responses. On the negative side — you reminded me of all the things I have forgotten or ignored whenever possible. grrrh!

    Patient, thoughtful responses to questions. Thank you. Whether people agree with you or not it is simply polite to recognize the effort.

  75. Got too many foolks in “climate science” that don’t get the distinction between “hypothesis” and “fact.”

  76. It has nothing to do with all the laws of physics, that is simply a distraction. We know that however the climate works, it works in accordance with the laws of physics. But there are very many ways for it to work in correspondence with the laws of physics. In some of those ways, the reaction to a small forcing will be to amplify it. In others of those ways, the reaction will be to damp it down.

    We need to treat the climate as a black box. If we apply a warming impulse to it, what actually happens? What has happened in the past, what is happening now? If we take away some of the warming input, what happens? Does it stabilize at cooler temps, warmer temps, or does it stabilize at the same temp?

    This is the core central question. Answer this, and you have the answer to the AGW issue. We know doubling CO2 produces a small warming input. But what is the long term effect of this and similar sized warming inputs? That is all that matters as far as the AGW hypothesis is concerned. Everything else is a distraction.

  77. I would add, this is like looking at a car, and trying to figure out how many miles it will get to the gallon from principles of physics. That is not what it depends on. It depends on how it is made. The only way to find out is to drive the thing and measure. Physics will give you some kind of plausible limits, you know how much energy there is in a gallon of gas, and how much it most likely takes to move the thing given rolling and air resistance, and inertia. But where it falls between those limits is how its designed, and depending on the variables of construction and design, a huge range of mileages is possible. Planck and Stefan-Boltzman are not going to help you with that.

    This is why AGW is not ‘just physics’. Its engineering.

  78. It’s a mean free path for radiation.
    And convection goes up.
    It works both ways with insulation.
    For every mother’s pup.

    Our conduction’s been declining-
    we were once a molten ball.
    So we might see another ice age
    after all.

    Stellar fusion bakes a planet orbitting a star.
    On a wobbly rotisserie, we’re undercooked so far.
    The hot spot mainly hits it on the rotating equator.
    Vapor rises, radiates and condenses elsewhere, later.

    The ocean sloshes heat around, and swaps with cooler areas.
    Water’s 3 phases play their thermodynamic stradivarius.
    Near the poles it doesn’t get the sun, so it gets cold, I’m sorry.
    Did you know your albedo shows out to Alpha Centauri?

    Oh there is no global warming.
    Even gormless should see.
    Oh there is no global warming.
    There is no free energy.

    We can laugh at those witchdoctors
    as they cry ‘catastrophe’.
    Because the universe will die of heat death-
    from ENTROPY.

  79. RE #32:
    The dominant reason an arid location has more violent diurnal air temparture transients than an equally located humid location is heat capacity.

    If I nuke this out too much, just let me know and I’ll try to downshift my explanation.

    Water (in all phases) is a wonderful heat storage medium. It soaks up a lot more energy than many other materials before it’s temperature changes (i.e., it has a relatively high heat capacity). The heat capacity, Cp, of water vapor is extremely high relative to almost all other natural atmospheric components.

    During the day, the air column is heated as the energy radiates into the air from the sun at a rate proportional to (Tsun^4-Tair^4). At night, the air column cools off as the heat radiates towards space in the night sky at a rate proportional to (Tair^4-Tspace^4). There are clearly other heat transfers (convective flow from surrouding areas, conductive flow from the ground) going on, and convective flow (weather) is often the primary driver. But weather aside, the energy loss to and from the air column for these two ideal sites (one dry and the other humid) will be about the same (though the absorption coefficient will be different since one site has a higher concentration of water).

    So given an equal amount of energy loss (or gain), the humid area will change temperature less because the water can ditch (or suck up) more units of energy without changing temperature as much. So in Honduras (15N, 85W) for example, you only get a 10-20 degree swing every day just about anywhere in the country. But in the souther Sahara (15N, 15W), you get huge swings in temperature between day and night. So it really is the “cold desert night” explanation.

    So even if the greenhouse effect exists, it clearly doesn’t overtake the more easily observed local temperature drivers. Maybe the greenhouse effect only acts on the average global surface temperature? 🙂

  80. 99, Dillon: I agree with most of what you say, but you just had to throw in the “cold desert night” myth. if you look at actual temperature data, you will see that (in the warm seasons) it is almost as hot in the desert during the night as it is in the humid areas during the day! The “cold desert night” myth only applies to HIGH deserts. But high “non-deserts” (such as Alamosa, CO) have the exact same patterns. See data for USA here: http://rredc.nrel.gov/solar/old_data/nsrdb/redbook/mon2/state.html

  81. Re: Jeff Id (Feb 4 23:36),

    Writing the comments doesn’t require all that much patience. It’s a useful exercise to put it down in print to see if it all hangs together. The patience I’m referring to is in politely replying to questions/comments despite what you might think about them. Read the comments to Willis E.’s The Steel Greenhouse and Willis’ replies. I couldn’t do that, or at least not for anywhere near as long as he did. After a while, my inner bender demands to be let out to play and I stop resisting temptation. That’s also why I was really happy to find killfile. Now if it would only work at The Blackboard. I’ve emailed the author of the script, but no reply so far. Anybody here know javascript?

  82. For those who think that because the center of the CO2 band is saturated at the surface adding more won’t make a difference, here’s something to think about. MODTRAN calculated radiant power 0 km looking up, 1976 standard atmosphere all other settings default: 375 ppmv CO2, 348.226 W/m2; 750 ppmv CO2(no, I don’t think we’ll actually get there), 349.796 W/m2. Not only is there less energy leaving from the TOA with an increase in CO2 (all other things being equal, which they probably wouldn’t be), but the surface sees a higher energy flux (higher sky brightness temperature).

  83. TOA is Top Of Atmosphere?
    LTE is Local Thermal Equilibrium?
    @104

    Speaking simplistically, what fraction of the CO2 population has already absorbed their IR photon on average?

    Why is is OK to use blackbody math in the situation of preferential absorption/emission?

    What is the mechanism of broadening of the CO2 absorption ‘line’?

    And, in the model of your choice, what would be the delta T at the surface for this change in radiant power and flux (given that all other things remain the same)?

    What magnitude of surface albedo change or cloud cover change would give a similar delta T, again with whatever model you choose?

    Sorry I have so many questions.
    TL

  84. #106, thanks for the link. Reading Held/Soden, on first glance, it looks like McIntyre asking for someone in climate science to publish an engineering-level, model independent, explanation of 3C per doubling sensitivity to be impractical. While on Page 448 is derived the feedback dependence on water vapor, Page 449 and 450 show that climate science today clearly says that water vapor feedback is derived from models with all of the difficulties of direct estimation of water vapor feedback strength described on Page 450. Therefore, the only method appears to be indirect validation by testing the validity of climate models.

  85. One big argument FOR AGW has been that “nothing but the increasing levels of CO2 could explain the increase in temperatures.” (regardless of the fact that this is an illogical argument). I wonder what these guys are attributing for the leveling-off and actual cooling over the past 12-15 years. Where are all those putative Watts of energy from the increased CO2 going? Do you think it is possible that there is something wrong with the GHG theory?

  86. Re: TurkeyLurkey (Feb 5 15:19),

    Yes on the meaning of TOA and LTE.

    The fraction of CO2 with sufficient energy to be in an excited state for emission at 667 cm-1 depends on temperature (~exp(-E/kT) where E is omega*c/h, omega is the frequency in cm-1, c is the speed of light in cm/sec, h is Planck’s constant, k is Boltzmann’s constant and T is the absolute or Kelvin temperature). At 288 K, it’s about 3.6% so 96.4% are capable of absorbing a photon at 667 cm-1. ‘Already absorbed a photon’ doesn’t really have much meaning as the system is dynamic with constant energy interchange by inelastic collision and radiation absorption/emission.

    The Second Law of Thermodynamics requires that nothing at a given temperature can emit thermal radiation (things like lasers don’t count as they are not thermal) at any wavelength that exceeds that emitted by a blackbody at that temperature and wavelength. So to obtain emitted radiant flux, one multiplies the emissivity (which has a range of zero to one) at a given wavelength or frequency by the Planck function (which describes the emission spectrum for a blackbody as a function of temperature) for that wavelength or frequency at the temperature of interest. Note that the Planck function looks very different for frequency vs. wavelength. That’s because the emission is watts per unit area (or in some forms per unit volume and also per unit solid angle or steradian) per unit wavelength or per unit frequency.

    Line broadening is caused by motion and pressure (collisions). Even at absolute zero, a line has finite width because of the uncertainty principle. Motion or Doppler broadening is because the effective frequency shifts with velocity (the train whistle thing). Since molecules in a gas have a distribution of velocities, you get a line shape that reflects this distribution. The area under the broadened line is constant (IIRC) and depends on the line strength. Doppler line width increases with increasing temperature and decreases with molecular mass. Pressure broadening is much more complex. It’s a function of gas density and temperature. The resulting line shape is approximated by the Lorentz distribution and the constants involved are determined by measurement rather than theory. The HITRAN database of molecular lines includes the Lorentz constants for each line. Pressure broadening dominates in the lower atmosphere and Doppler in the upper atmosphere. The Voigt line shape, a hybrid of Doppler and Lorentz, is used in the transition region.

    For calculating delta T for a given change in CO2 (or methane, water vapor or ozone) to a first order approximation, you can input a temperature offset in the Archer MODTRAN page to restore emitted power at the TOA to the original power. You can include some feedback from water vapor by setting the ‘hold water vapor’ to Rel. Hum. instead of pressure. Example:

    1976 standard atmosphere, constant pressure, 100 km looking down.

    CO2 280 ppmv Iout 259.992 W/m2, ground T 288.2 K
    CO2 560 ppmv Iout 259.992 W/m2, ground T 289.07 K, delta T 0.87 K

    Constant Relative Humidity
    CO2 280 ppmv Iout 259.992 W/m2, ground T 288.2 K
    CO2 560 ppmv Iout 259.992 W/m2, ground T 289.49 K, delta T 1.29 K.

    The equivalent change in albedo isn’t very large. For the constant relative humidity case, the change in emitted power at constant temperature is 2.863 W/m2. Assuming 235 out of 342 W/m2 is absorbed now, the albedo is 0.3129. So to reduce the absorbed power by 2.863 W/m2 the required albedo would be 0.3212 or a relative increase of ~1%.

    These are crude calculations, but they should be in the ballpark, neglecting other feedbacks which may or may not be present.

  87. On further reading, it looks like confidence in a minimum 1.5C per doubling is very good, ‘robust’ so to speak i.e. Beta_H20 ~ 0.4.
    Page 455
    Because cloud and water vapor feedbacks are obviously related at some level,
    they are often confused in popular discussions of global warming. In the current
    generation of climate models, water vapor feedback is robust and cloud feedback is
    not. A robust water vapor feedback sensitizes the system, making the implications
    of the uncertainty in cloud feedbacks of greater consequence.

    They derive this estimate on Pages 462/463.

  88. RE 46.

    A while back bender and I kind of stumbled on this same point in discussing the tenets of Lukewarming.

    My basic position was that feedbacks were unlikley to be a net ZERO and more probably a net positive.

    That was based on current estimates of the feedbacks and uncertainties. So that the Lukewarmer position was, on my view,
    committed at least to a “net positive” position.

    bender, made a wise comment about time scales, I punted. scatched my head and punted again.

    WRT your position: I have a really hard time placing you, vonk, sometimes bender. Vonk especially since his position seems to go to the heart of understanding itself.

  89. Re: RB (Feb 5 17:47),

    To say cloud feedback in the models is not robust is something of an understatement. My impression is that the models don’t get either the location or amount of cloud cover anywhere near correct. Location is important because whether clouds are net negative or positive feedback is dependent on location. IIRC, the feedback effect of clouds at high latitude is positive and negative at low latitude.

    Then there are aerosols and specifically the aerosol indirect effect. Aerosol dependence seems at first glance to be primarily a tool for tuning the model response so it fits the instrumental record. One suspects this because the sensitivity to CO2 and aerosols within models is highly correlated.

  90. #112 DeWitt Payne,
    My understanding was that whether clouds are net positive or negative feedback was dependent on height, no? Low thick clouds = negative, high thin clouds = positive?

  91. Re: steven mosher (Feb 5 18:18),

    I disagree that believing in net negative feedback means one isn’t a lukewarmer. That would rule out Lindzen for one. Negative feedback doesn’t mean no warming, just less warming. So IMO a lukewarmer is anyone who believes that doubling CO2 will produce a detectable change in temperature.

    As far as Vonk, at one point a while back he seemed to be a G&T’er. I had some discussions with him over at UKWeatherworld that gave me that impression. Some recent comments of his lead me to believe that he has moved on from that position. Now Vonk might not be a lukewarmer because he doesn’t believe that we can reliably detect even a fairly large change in temperature as a result of the chaotic nature of climate.

    I think we’ll have a much better understanding after a few more years or ten of ocean heat content data acquisition. That’s a direct measure of radiative balance.

  92. Re: RB (Feb 5 18:33),

    The altitude and composition of clouds is indeed important. Cirrus clouds are definitely positive. The ice crystals that form cirrus clouds have significant reflectivity in the thermal IR. I should look this up, but I’m pretty sure there was a measured drop in temperature for the days after 9/11/2001 when there were no jets in the stratosphere over the Arctic making contrails, which are effectively cirrus clouds.

  93. I think Mosher #111 has a definition of lukewarmer as

    dT = 0.9C/(1-f)

    Lukewarmer f>0, dT>0.9C

    Lindzen = -infinity<f0.25)

    IPCC range 0.4<f<0.8

  94. Oops that got messed up with the left brackets
    lindzen = -infinity less than f less than 0 but the Trenberth fix to the most glaring error in Lindzen/Choi gives dT > 1.2 and f>0.25

  95. Well, and here Eli goes back to the original post, to start with the way you state the problem is incredibly awkward, which is a sure sign that you need to pick up a simple book on atmospheric science. David Archer’s “Understanding the Forecast” is a good place to start, there are a bunch of applets that you can fool around with to get a good feel of what is happening and video lectures which are free to listen to although slow to download.

    A nit pick on #109 – there are two equivalent (aka degenerate in physics speak) CO2 bends, so you have to double the 3.6% in v=1.

  96. #121, Thanks for pointing me to an elementary book on warming video lectures on radiation absorption. Very useful. I suppose it’s time to quit running my optics company now.

  97. Re: Eli Rabett (Feb 5 20:19),

    Thanks for the correction. I saw the degeneracy term in the equation and knew that there were in-plane and out-of-plane modes, but failed to make the connection when I did the calculation. I’ll be really surprised and pleased if that’s the only mistake I made.

  98. “I should look this up, but I’m pretty sure there was a measured drop in temperature for the days after 9/11/2001 when there were no jets in the stratosphere over the Arctic making contrails, which are effectively cirrus clouds.”

    Might be a different study, but I saw one reporting a slightly over 2-sigma low temperature over the continental US for those days, although it has been noted it was generally cold weather then anyway.

  99. #116, #125,
    That is actually very surprising because those cirrus clouds are supposed to have a warming effect, per Caballero lectures Page 83 .

    A more dramatic example of the same phenomenon are contrails
    (short for condensation trails), the linear clouds stretching behind aeroplanes high in the
    sky (Fig. 3.13). Contrails are of interest to various people, among them the Air Force: it
    makes the generals look bad when their multi-zillion dollar stealth fighter has a large cloud
    pointing at it like a neon sign. A more PC interest in contrails derives from their possible
    role in enhancing global warming. If you’ve ever stopped to look at contrails for a while, you
    will have noticed that often they don’t just disappear, but evolve into more horizontallyextensive
    cirrus clouds. These clouds are thin enough to let plenty of sunlight through, but
    they still trap infrared radiation quite strongly (again, more on this later), so they lead to a
    net warming.

  100. 116, 125, 127-I happen to have just been looking, by coincidence, for a paper on whether the air traffic change detectably changed the US Diurnal Temperature Range. The conclusion was that it did NOT have a detectable effect, contrary to other analyses:

    Click to access c026p001.pdf

    Of course this is in the taken over journal and at least one author is a dreaded skeptic 😉

    Whatever the case may be, the IPCC puts contrails at a very small contribution to the forcing.

  101. Here is the arxiv link.

    Interglacials, Milankovitch Cycles, and Carbon Dioxide
    Authors: Gerald E. Marsh
    (Submitted on 2 Feb 2010)

    Abstract: The existing understanding of interglacial periods is that they are initiated by Milankovitch cycles enhanced by rising atmospheric carbon dioxide concentrations. During interglacials, global temperature is also believed to be primarily controlled by carbon dioxide concentrations, modulated by internal processes such as the Pacific Decadal Oscillation and the North Atlantic Oscillation. Recent work challenges the fundamental basis of these conceptions.

  102. The Soden and Held paper reminded me that not all atmospheric IR absorption/emission is the result of the line spectra of ghg’s. There’s also continuum absorption. This results from a mechanism that seems to be similar to collisional line broadening and is called collision induced absorption or CIA for short. The collision distorts the electric field of a molecule and creates a dipole where there might not have been one before. As a result, even nitrogen has some continuum absorption in the low frequency end of the thermal spectrum with a peak at about 100 cm-1. CIA for N2-N2, O2-O2 and N2-O2 collisions is weak in the thermal IR and is only important for paths that don’t intersect the lower atmosphere, i.e. where water vapor concentration is insignificant. It may be more important for absorption of incoming solar radiation in the near IR. H20-H20 collisions and the resulting continuum absorption is many orders of magnitude stronger than collisions involving N2 and O2 and so is important and must be used when calculating atmospheric spectra in the troposphere. The theory isn’t all that good so the absorption parameters are measured empirically. The fit is pretty good, though, so the difference between calculated and measured absorption and emission is small.

    Also, the oxygen molecule has a non-zero magnetic dipole moment so it does have rotational bands in the microwave region. The wing of the band at 60 GHz is used for remote sensing of atmospheric temperature by satellite. This is the reason for the name (A)MSU or (Advanced) Microwave Sounding Unit. This leads to a certain irony when people who don’t believe in the greenhouse effect use satellite temperatures to advance their argument. The theory behind remote sensing of temperature by satellite is on the same foundation as that for the effect of IR active molecules on the atmospheric temperature distribution and surface temperature.

  103. Hmmm, no answer to 108, which is repeated here:

    One big argument FOR AGW has been that “nothing but the increasing levels of CO2 could explain the increase in temperatures.” (regardless of the fact that this is an illogical argument). I wonder what these guys are attributing for the leveling-off and actual cooling over the past 12-15 years. Where are all those putative Watts of energy from the increased CO2 going? Do you think it is possible that there is something wrong with the GHG theory?

    The heat is evidently not hiding in the oceans:

    http://wattsupwiththat.com/2010/02/05/more-on-ocean-heat-content-and-recent-revisions-to-the-data/#more-16094

    Where are the Watts?

  104. More words of wisdom, before I run out of beer: Most of the “true AGW believers” don’t seem to have a clue about what science is. Equations and MODTRAN models, like those championed by DeWitt Pain and , don’t prove a damn thing. The proof of the pudding in science is EMPIRICAL EVIDENCE. Einstein’s theories were not proven, until experiments showed he was correct. Empirical evidence is totally missing in the AGW CRAP. Indeed, the evidence seems to now discredit it. And that is why it is NOT good science.

  105. Despite DeWitt Payne’s erudite explanation of the mechanism of GHGs, we still need to explain the poor correlation between [CO2] & temps over the past century or so. The differential spectra of incoming vs outgoing radiation for the planet may quantitate the energy balance, but do not account for that poor correlation.

    What portion of the heat content of the atmosphere is gained by absorption of EM radiation vs that gained by direct conduction from the surface?

  106. #34

    De Witt, you stated two of my favorite ironies very well.

    1. Imagine no atmosphere, like the Moon. A little hotter, a little cooler with no atmosphere? (forget H2O)

    2. The notion of proxies. MSUs, etc., are proxies too. Do I like satellite proxies (assuming proper calibration) or do I like tree-ring proxies no matter how well they are ‘calibrated’?…satellites, imo.

  107. #122

    The explanation in the video is the same as has been presented here several times in the past, and as is described (more briefly) in Soden and Held. If you don’t have much time, you could set the computer to download the one for the second half of chapter five, and later when you have a spare half hour, have a look.

    But it just explains more slowly what has already been explained.

    Convection combined with the moist adiabatic lapse rate sets a vertical temperature gradient of 6C/km. Radiation to space occurs at a temperature to balance the incoming heat from the sun of about -24C. Because some of this radiation is emitted at altitude, the average altitude of emission is above zero, and there is an adiabatic temperature difference between this altitude and the ground, that warms the ground to about 30C above that temperature. Adding more greenhouse gases raises the average altitude of emission by 160m/2xCO2, raising the temperature about 1C/2xCO2.

    The Greenhouse Effect is a real physical effect, that does contributes a small warming, but it doesn’t work by ‘trapping’ radiation.

    And all the arguments about the layer model, and details of emission spectra and the quantum molecular dance, either to get numbers to fit or to prove that no such numbers can be provided, are all besides the point.

    Once you understand the basic claim of the AGWer climatologists, then you can start questioning whether the lapse rate is constant everywhere and all the time, what effect local changes in humidity have, how clouds and terrain and latitude affect it, whether you can legitimately average things in that way, and about the horizontal transfer of heat by Hadley cells and ocean currents. There’s a lot to argue with.

    But the real debate to be had gets drowned out by the noise.

  108. #138, I assure you again that I understand the radiation aspects of global warming. It’s really frustrating having people assume that I don’t. The mechanisms are obvious to those of us who have worked with the math. I don’t believe the 30 degree number is perfectly accurate despite having calculated it myself on this blog due to some assumptions about the absorptivity of the earth on average.

    Also, my use of the word trapping isn’t as far off as you (and many others) claim, there is a steady state increase in heat caused by the shortening of the path length of IR. This portion of heat is trapped in the sense that although it’s in continuous flux, the total amount is higher. So once you understand that I understand, we can go further discussing.

    Oddly though, the point people missed here when referencing me was that others were having this discussino on a different thread. I wanted to move it here so I wrote my opinion quickly and let it rip. As I said in the headpost, I really find little new info in any of this discussion. DeWitt has a nice comment above that caught my attentino but a lot of the rest is without quantitative interest.

    I don’t know everything but let’s not imagine that an Aeronautical engineer who spent his career consulting and working in various aspects of optics and an entire year and thousands of hours blogging on it hasn’t figured out the basics. That is TRULY annoying and a little lame.

  109. Please excuse my ignorance, but I don’t think the ElephantMan looks anything at all like an elephant either.

    In the atm emmission spectra sited in post 63 above http://i165.photobucket.com/albums/u43/gplracerx/PettyFig8-2.jpg the EM from the sun, filtered by the atm on the way in, seems to allow all energy of wavelength ~600-750 thru (fig b), but then ~1/2 of the energy radiated by the earth at those same wavelengths seem to be absorbed on the way back out (fig a). Apparently this is where CO2 is performing its GHG duty.
    Apparently the intercepting CO2 molecules know which way the radiation is flowing.
    Like the Thermos bottle that keeps hot things hot & cold things cold: How do it know?

  110. Jeff ID, I think you have an appreciation for and understanding of the physics of CO2 warming (without feedback) and I would judge that most others think so also.

    Were I think we can waste a lot time is when laypersons attempt to “make” up their own theories and models of how GHG affects GW and then we get these discussions that often end up with somebody getting hurt feelings.

    I think what Binned is saying, and DeWitt has said in the past, is that there is plenty to discuss about the science (and the controversy that it can involve) that can be found explained in the the mainstream literature without getting involved in discussions of half-baked pet theories.

  111. Kenneth, 141 and others:

    Steve McIntyre has been asking for years for some type of good explanation of how the AGW theory works, physics-wise. So far no luck, AFIK. So, just what does the “consensus” believe?

    There are quite a few very well-regarded physicists, like Gerlich and Tscheuschner, Miskolsczi, Schreuder, Thieme, etc. who disagree with whatever passes for most folks as the basic physics behind AGW (of course, we don’t know exactly what this is, as mentioned above).

    The empirical evidence seems to indicate that something is wrong with the Held/Soden 2000 ideas: there is no “hot spot” in the mid troposhpere in the tropics, and the water vapor levels are not increasing.

    Simple observations show that just increasing the amounts of GHGs does not increase the temperature. Again, compare Phoenix to Miami.

    The physics is not “settled,” and there is plenty of room for new ideas, even if they are “half-baked pet theories.”

    Those who think these discussions are worthless don’t have to read this thread.

  112. A few more questiones for Dewitt;

    What, in your opinion, is the ‘mean free path’ for LWIR photons emitted from the earth’s ‘surface’ before their seemingly inevitable capture by those ~rare, yet rapacious, unexcited CO2 molecules?

    I apparently said something improperly about the 3.6% of the population which are incapable of absorbing a photon for being in the wrong state. I suggested that this was because they already had done so, in a simplistic single molecule/single photon description to a statistical equilibrium situation.

    And in a very different genre, how do physically enlightened persons think about the boundary layer which I imagine to cling to the earth’s surface? I’m trying to integrate this into my so-called ‘thinking’, particularly at night when minimum daily temperatures are being established and the external incident flux is relatively negligible. I have the idea that the historic instrumentation base is sometimes within a ~static ‘boundary layer’, and then not so during sufficiently windy conditions?

    And in yet another genre, what flux (if any) of internal ‘heat’ do physically enlightened persons attribute to heat of subsurface ‘origin’ ? Do reputable enlightened persons still refer to a nuclear reactor at earth’s core, or do gravitational tidal effects play a role? It is important to remember that Linda Morabito (who first observed the purported vulcanism on Io), was Not a member of the physically enlighted persons society.

    And returning to the original question, is it correct to think that the extrinsic flux at the high-energy tail of the distribution is embodied within the nominal W/m2 numbers quoted? I’m a bit puzzled how the solar ‘flares’ seemingly come and go without some net change in the perceived terran flux. Do solar ‘flares’ represent variation in the color temperature of the sun, or merely unseemly deviation from black-body models? I had the impression that solar ‘flares’ were like burps of high energy photons/particles, that did not correspond to a visible shift in the solar emission.

    Pardon me in advance if this is the post that pushes you over the Willis SteelShell brink of absurdity…

    And when I bake my pets, I’m always careful to set the oven for Full Broil… no half measures here…
    TL

  113. #139. I assure you, I do quite believe that you understand the radiation aspects. What I’m not yet convinced you understand, because you haven’t mentioned it in any of your explanations, is the essential role of the lapse rate.

    I can’t think of a good analogy – the best I can come up with is that it is like arguing that the inside of a refrigerator is cold because of the thick thermal insulation in the walls. The heat is ‘trapped’ outside the box.

    It’s not that it doesn’t happen, or has nothing at all to do with the end result, but it doesn’t properly *explain* the cold, and it doesn’t explain why the inside is at the temperature it is at. Add more insulation, and the motor will do a little less work while the temperature inside stays constant, because it is the thermostat that controls temperature. And the reason CO2 makes a difference is not because it thickens the insulation (although it does), but because it changes the location of the thermostat.

    I think perhaps you assumed that the lectures Eli pointed you to were another basic introduction to the radiative balance argument, that you already know because it’s presented everywhere. It’s true that David Archer starts with this, but then he tells you that it’s a case of getting the right answer for the wrong reason, and goes on to explain about the lapse rates. (Chapt 5 Pt 2.) It’s quite apparent from this that all the climatologists *know* how it really works. So why you still get the likes of this rubbish in public presentations is an abiding mystery to me.

    All the arguments about heat flows through the refrigerator walls, and whether the insulation can possibly work at the quantum level, or whether refrigerators violate the laws of thermodynamics…, they’re all nugatory, because everybody is off chasing after this bogus explanation, while the real one gets away.

    I find it hugely frustrating myself. I agree that the 30C number is likely inaccurate, and I think there are still huge problems and gaps with the explanation even as I’ve given it. But as David Archer said in his lecture, radiative balance alone predicts a change in temperature with altitude of 16 C/km, and that is contradicted by observation. To quote Feynman: “It doesn’t matter how beautiful your theory is, it doesn’t matter how smart you are. If it doesn’t agree with experiment, it’s wrong.”

  114. I am sorry I came late, but I would like to point out something that appears to have been missed. The planet Earth rotates. At dawn it begins its heating cycle and any point on the surface reaches maximum temperature between noon and 3:00; then it cools.
    Most of the cooling occurs during the night.
    Modeling the average temperature from max-min/2 and the heat input from total W/m2/24 hr is not only pointless it is stupid.

  115. Re: guidoLaMoto (Feb 6 16:52),

    Those spectra, which are emission, not absorption (the y axis scale is power per unit area, solid angle and frequency unit, or milliwatts per square meter steradian reciprocal centimeter, not percent transmission, absorption or absorbance) only cover part of the thermal IR range (400 to 1700 cm-1), which is considered to be from about 100 micrometers to 4 micrometers wavelength or 100 to 2500 reciprocal centimeters (cm-1), which IR spectroscopists use instead of frequency (the scale is more convenient than GHz to THz). The solar spectrum has most of its energy in the range from 4 to 0.2 micrometers (2500 to 50,000 cm-1). So the spectra say nothing about incoming or reflected solar radiation.

  116. Re: Binned (Feb 7 07:28),

    The radiation calculations assume that the lapse rate and humidity and other ghg profiles are known. Why the lapse rate is what it is at any given point on the surface and the interaction between radiation and convection is another topic entirely and is about as complicated as the radiative calculations alone. I could have made a stab at that too, but it didn’t seem to be the main topic of the thread. It’s discussed in detail (lapse rate and humidity anyway) in the link in comment #86 above.

  117. #147. Agreed. That’s why it takes a full atmosphere/ocean model to even attempt it, and I don’t believe for a second that trying to solve such equations with grid resolutions of hundreds of kilometres has a hope.

    Mine is a toy model to try to explain the concept. It shouldn’t be taken to be anything more.

  118. DeWitt, I realize those are emission spectra, but is that not the complement of absorption? If the one is emission “looking up”, is that not tantamount to incoming solar radiation that is not absorbed by the atmopsphere, and “looking down” tantamount to surface re-radiation not absorbed by the atmosphere before escape?
    This gets back to my other question: in discussing the importance of GHGs in warming the atmosphere, how much of the energy contained in the atm is actually derived from absorbed IR vs direct conductance from the surface? I should think the overwhelming amount comes via the latter, judging by the success desert lizards & beach bums have in cooling their feet by alternatley lifting one at a time from the hot sand. Doesn’t this fact really minimize the “GHG effect” which may be qualitatively accurate on the quantum mechanical level, but quantitatively unimportant on the practical level?

  119. Re: guidoLaMoto (Feb 7 22:33),

    If the one is emission “looking up”, is that not tantamount to incoming solar radiation that is not absorbed by the atmopsphere, and “looking down” tantamount to surface re-radiation not absorbed by the atmosphere before escape?

    No, definitely not for looking up, somewhat true for looking down. Remember, the atmosphere emits as well as absorbs. In fact there cannot be absorbtion without emission as well (Kirchhoff’s Law of thermal radiation). Of course, you don’t get measurable emission in the visible and UV at atmospheric temperatures. These spectra were either taken at night or aimed well away from the sun. So looking up, the background emission is effectively zero. Looking down, the background is the thermal radiation emitted by the surface. Except for the ozone band at about 1000 cm-1, the atmosphere is nearly transparent between 800 to 1300 cm-1 for low specific humidity as is found over the polar ice sheet where the spectra I posted were taken. The brightness temperature looking up in that spectral range is about 160 K (compare the intensity to the Planck curves for different temperatures overlaid on the plot). That’s a lot less than 6,000 K for sunlight. Looking down, the brightness temperature is slightly less than 270 K in that same spectral range. That would seem to be a reasonable temperature for ice. Surface reflectivity in the thermal IR is very low, so even if the sun was up when the spectrum was taken, you’re still looking at surface emission, not reflected sunlight.

    For your other question, I refer you to the Kiehl and Trenberth energy budget. The transfer of heat from the surface to the atmosphere by convection is 102 W/m2. 78 W/m2 of that is by latent heat transfer. With no liquid water at the surface, like dry sand, the result is a higher surface temperature on a clear day. That diagram refers to a surface temperature of 288 K. At higher temperature the numbers would be somewhat different, but the principle is still the same. The ‘greenhouse’ effect is most important at night anyway. If the brightness temperature of the atmosphere were effectively zero, it would get a lot colder at night. The surface temperature drops a lot more on a clear winter night than it does on a cloudy night (outside the tropics anyway) because low clouds have a brightness temperature much higher than the clear winter sky. The difference can be on the order of 100 W/m2.

  120. Re: TurkeyLurkey (Feb 6 21:05),

    I don’t have the mean free path number off the top of my head. I could look up the formula and probably calculate it, but I’m curious as to why you want to know. It’s going to vary a lot with wavelength, short where absorption is high and very large where it’s low. At low specific humidity, it’s large enough in the 800 to 1300 cm-1 that a large fraction of emission escapes directly from the surface to space. At the very center of the CO2 band at 667 cm-1, it’s probably on the order of centimeters.

    You can google ‘planetary boundary layer’ and get lots of hits (Wikipedia entry for one).

    The contribution to the surface temperature from heat leaking from the planetary core is tiny when averaged over the whole surface. It’s way less than 1 W/m2. Obviously, it’s a lot higher in some spots than others.

    The large variability in solar emission at the short wavelength end of the spectrum may well have a significant effect, but nobody really knows right now, as far as I can tell.

  121. This whole business about GHG’s re-radiating is trivial. Temperature is a measurement of the average kinetic energy of molecules. When a GHG absorbs IR from the Earth’s surface it instantly gains kinetic energy, and Brownian motion quickly transfers that energy to the adjacent molecules, raising the average kinetic energy of the atmosphere. We don’t speak of gas re-rediating internally as it must average to zero heat transfer, we only speak of the flux, the heat transfer from the surface of the gas, either to the Earth or to space. The Earth definitely loses heat to space at night. Water can freeze at 59 F on a clear night with no wind, documented many times.
    The greenhouse effect is poorly named but real. The question of saturation is very important. With CO2 at 390 ppm, we should be seeing warming instantly, not over time. Since we aren’t, the probablility of saturation looks most likely very high. Why was NASA-GISS’s warmest year 1934 published in 2005, revsed to be 1998 in 2007? Because in 2007 they decided they wanted it to be that way!

  122. Thanks for your patient answers, DeWitt, I hate to be a PITA, but:
    Kiehl & Trenberth’s energy budget doen’t make sense. How does the 168W/sq m of solar energy absorbed by the surface, with 102W/sq m then lost to the atm as thermals & evaporation, suddenly become 390 W/ sq m of surface radiation? If the GHGs are “back radiating” 324 W/sq m towards the surface (where’d they get it?), then they must be radiating that much outwards also- but that is not taken into account in the figure.
    Then we see 30W/sq m mysteriously leaving the cloud on the egress side, without explanation, and that leaves us with 27W/sq m unaccounted for out of that unexplained 390.

    It would seem that, of the 342W/sq m solar radiation not reflected, 168W/sq m ends up in the atm (as 169W/sq m by their count), and 165 of that is joined by the 30 mysterious cloud Ws and another 40 escaping via their “atm window”, so the total energy lost to outer space is 235W/sq m. In other ewords, the planet should be cooling.

    That 390W/sq m seems to be in some GHG cycle with the 324 W of back radiation. And that cycle would appear to be necesarily a diminishing spiral if 390 W turns into 324w. No?

  123. Guido, Eli has something on that Also, contrary to what has been said, the MODTRAN simulations have been evaluated against observations and shown to be accurate and precise.

    Michael Moon is not quite getting it. The ghg absorbs the photon and is vibrationally excited. This excitation degenerates to kinetic energy during collisions with other molecules such as O2 and N2. OTOH, kinetic (thermal energy) can vibrationally excite ghg molecules by collision. The net is that there is an equilibrium population of a few percent of vibrationally excited molecules, and the ghg molecules that are excited can emit radiation to space and the surface and other ghg molecules in the atmosphere.

    Rabett Run will probably have a post on the water vapor continuum in the next week.

  124. Re: guidoLaMoto (Feb 8 19:12),

    Ok, lets go over the diagram piece by piece. First there’s incoming soar radiation at 342 W/m2. 77 W/m2 is reflected by clouds and other aerosols and particulates. 30 W/m2 is reflected by the surface. That leaves 235 W/m2 to be absorbed. 67 W/m2 is absorbed in the atmosphere. That’s mostly in the near IR and the UV. The UV absorption is almost entirely in the stratosphere as a result of oxygen absorption at very short wavelengths. The excited oxygen then can form ozone, which absorbs somewhat longer wavelengths in the UV. As a result, not much radiation shorter than ~0.3 micrometers reaches the surface. Near IR absorption is mostly water vapor and clouds. That leaves 168 W/m2 absorbed by the surface.

    Now for outgoing. The surface is at 288 K and the emissivity is very close to 1.0 so emission of 390 W/m2 is required by thermodynamics. Of that 390 W/m2, 40 W/m2 escapes directly to space, the remainder is absorbed by the atmosphere. In addition, the surface transfers heat to the atmosphere by convection at a rate of 102 W/m2, 24 W/m2 by air that is warmer than the air around it, called sensible heat, and 78 W/m2 by the evaporation of water from the surface and condensation in the atmosphere, called latent heat.

    So the atmosphere absorbs 350 plus 102 plus 67 W/m2 for a total of 519 W/m2. That heat has to go somewhere or the atmosphere would heat up. Also the total emission to space must equal 235 W/m2 or the system would heat up. Since 40 W/m2 is going to space already from the surface, that leaves 195 W/m2 to be emitted from the top of the atmosphere by the atmosphere itself. Subtracting 195 from 519 leaves 324 W/m2 emitted toward the surface. This is possible because the temperature of the atmosphere goes down with altitude and is optically thick over wide ranges of the emission spectrum. As I pointed out above, emission is a function of temperature so the warmer lower atmosphere emits more than the top. You can calculate this by cutting the atmosphere into layers and it really does work. Each layer emits the same amount of radiation up and down, but each layer is colder and absorbs and emits less radiation than the layer below it. This is why the analogy of the greenhouse effect as insulation has validity. A blanket also has a temperature gradient and emits less radiation to the room than it does toward you. Resistance to heat transfer results in a temperature gradient when there is heat flow just as an electrical resistor has a potential gradient when current flows. Resistance to heat transfer is the definition of an insulator.

    So now we look at the energy balance for the surface. The surface receives 168 W/m2 from sunlight and 324 W/m2 from the atmosphere for a total of 492 W/m2. It than emits 390 W/m2 as radiation and transfers 102 W/m2 to the atmosphere by convection. Those are gross figures. The net transfer is 390-324 or 66 W/m2 by radiation and 102 W/m2 by convection. Those total to 168 W/m2, precisely the number the surface receives from the absorption of sunlight. So the numbers do add up as expected.

  125. Folks, the K&T “energy balance diagram” is just plain wrong. It supposes an “average” balance, which is stupid. Take an individual location and do the calculations for high noon in, say, Atlanta. You will find that the combination of “backradiation” and direct solar radiation would frigging COOK us!

  126. Time out: you still haven’t explained how only 342 W/sq m entering the top of the atm, with a good chunk of it reflected back without doing any warming at all, suddenly turns into 390W/sq m radiating off the surface, even after giving up 102W/sq m? That would have to be some magic heat engine you have there. Not to mention how does the cooler 324W/sq m get abosrbed by a surface that’s radiating at a warmer 390 W/sq m? The PhysicsPolice shoud be writing all sorts of violations here. Are you suggesting the surface gets to be 288K by virtue of geothermal energy and not from solar radiation? What would be the temp of the planet surface if there were no Sun?

  127. PS/ These energies we’re talking about are reported as “rates,” but W/sq m is volume. Aren’t we missing the time factor here? Isnt that reaally what’s important? The question is really does the atm have time to cool down each nite after warming each day?

  128. Eli, one of us isn’t quite getting it. The Earth’s surface radiates in the infrared, simple. GHG’s absorb some of this, simple. What is not simple is your contention that GHG’s re-radiate back to the Earth. The only number that matters is the flux, which is the net heat transfer, in this case Watts per square meter(not volume by the way.) The GHG’s re-radiating back to Earth surface is trivial, as it is deducted from the net flux in the first place. GHG’s re-radiating to space? Why is such a small fraction of the atmosphere being so closely considered? The only important thing they do is absorb infrared and add heat to the atmosphere. The energy balance must balance, duh. Sunlight is 1366 W/m2 average daytime Earth’s surface, BTW.

  129. Michael Moon is right-on, IMHO.

    “Energy balance diagrams, such as the K&T diagram, are meant to represent an AVERAGE radiation balance for the whole earth. As I said above this is crazy. To demonstrate that, let’s do a thought experiment and look at a specific place and time of day. Let’s pretend that we have a greenhouse (to block convection) in Atlanta, GA. It is constructed of a material that is transparent to visible and IR (such as NaCl). Inside the greenhouse is a thin black horizontal plate. It is July. At noon, the direct solar radiation is something like 900 wm-2 and the air temperature is 30 C. The “back radiation” consistent with the outdoor air temperature of 30 C would be equivalent to a blackbody radiating at 478 Wm-2. Thus, according to the concepts put forth by the radiation cartoons, he total radiation striking the black panel would be the sum of 900 + 478 = 1378 wm-2. That is equivalent to a plate temperature of 122 C (251 F). It just doesn’t happen, even in a greenhouse. You just cannot add in the backradiation to get an energy balance.

  130. #160 Michael Moon:

    Hence the average incoming solar radiation, taking into account the angle at which the rays strike and that at any one moment half the planet does not receive any solar radiation, is one-fourth the solar constant (approximately 342 W/m²).
    http://en.wikipedia.org/wiki/Sunlight

    If you force a 1A current through a 1 ohm resistor, the potential difference is 1V
    If you force the same current through a 2 ohm resistor, the potential difference is 2V.
    The greenhouse effect is as simple as that if you blackbox the atmospheric resistance without caring about the internal physical processes.

  131. JAE,

    On a sunny summer’s day in the tropics, leave a thick matte-black metal plate in your car on the dashboard, so it can enjoy the mid day sun. When you get in a few hours later, try to pick it up. And that’s with a limited amount of convection, conduction, etc. still going on inside the car.

    But I’d argue that the problem with those energy budgets is that everybody is assuming the radiation flux determines the temperatures. I suggest to you that it is the temperatures that set the radiation fluxes. It gets causality the wrong way round.

  132. Binned, 164:

    “But I’d argue that the problem with those energy budgets is that everybody is assuming the radiation flux determines the temperatures. I suggest to you that it is the temperatures that set the radiation fluxes. It gets causality the wrong way round.”

    EXACTLY what I think and have been saying for years!!! It’s all about heat storage, not radiation!

  133. While we are discussing energy budget, I had a brief email exchange with Trenberth, and while leaving the energy imbalance as an open problem he pointed to the von Schuckmann 2009 paper summarized below with measurements going down as deep as 2km as pointing to the need for observation systems in the deeper part of oceans (his ‘travesty’ comment was made prior to this paper):

    http://www.skepticalscience.com/global-cooling.htm
    http://www.skepticalscience.com/Understanding-Trenberths-travesty.html

  134. “..everybody is assuming the radiation flux determines the temperatures”

    ..and those everybody would be right. Equivalently, the sun is a current source, not a voltage source.

  135. Re: guidoLaMoto (Feb 9 07:47),

    Not to mention how does the cooler 324W/sq m get abosrbed by a surface that’s radiating at a warmer 390 W/sq m? The PhysicsPolice shoud be writing all sorts of violations here.

    Please (re-)read my post #74 in this thread where I explained exactly that. As I said there, that topic is not a subject for debate and I will offer no further explanation. You can also read the comments on The Steel Greenhouse.

    That knock on your door is the Physics Police coming to get you not me.

  136. Regarding #162, Trenberth made another important point on how a pure atmospheric resistor model is inadequate – because of the role of moisture in the atmosphere. Only some of the radiation is used to drive up the surface temperature, while some of it is stored as latent heat in atmospheric water vapor etc. etc.

  137. Re: RB (Feb 9 17:31),

    The deeper the ocean layer that contains the imbalance, the smaller the temperature change (we’re talking a few millidegrees for a 2 km layer), the harder it is to measure and the longer the equilibration time. If the equilibration time is a thousand years or more, who cares? The fossil fuel era will be over long before then.

  138. I still have a problem with the Steel Greenhouse analogy: if the steel shell reaches thermal equilibrium with the energy source (the planet’s surface in the example), why would it be radiating “back?” Doesn’t thermal equilibrium mean no net flow of energy between source and recipient?

    Would a better analogy involve a solid metal “atmosphere”- let’s just consider a metal column for simplicity, with some particular thermal conductivity value? Energy would then be conducted from the surface, thru the column, then radiated into space at the particular rate. Now change the composition of the metal (analogus to varying the [CO2] in our atm), and thus the thermal conductivity value, and then the rate at which the planet loses energy would change commensurately.

  139. Re: guidoLaMoto (Feb 9 20:40),

    Did you read the comments as well as the article? By and large, Willis answers your objections in detail in the comments. But I’ll give it one more try. As I said, I don’t have Willis’ patience.

    For one thing, it’s not really at equilibrium. There’s a constant flow of energy. For another, ALL objects that aren’t perfectly transparent or reflective and are above 0 K radiate. The intensity of that radiation is proportional to temperature It doesn’t matter whether there is something next to them or not. And finally all objects that aren’t perfectly reflective or transparent absorb radiation that impinges on them. That is NOT a function of temperature (to a first approximation anyway), although it may be a function of angle and wavelength.

    Now consider a container with no gas inside it to conduct heat. The walls of the container are maintained at temperature Ta. Now put an object with no internal heat source in the container with temperature Tb. It doesn’t matter whether Ta is higher or lower than Tb. What happens to the temperature of the object and why?

  140. DeWitt says he can’t see this comment (which is probably hokey-baloney), but I do believe that he is engaging in pure strawmanship in his latest post. It has absolutely nothing to do with the discussion of backradiation, global warming, CO2, GHGs, or our temperature, as far as I can tell….

    But… maybe I’m wrong. And I do appreciate your comments, DeWitt; and I enjoy them!

  141. All objects may radiate, but once in thermal equilibrium, net emission is balanced by net absorption. Your object with temp T(b) placed in the container of temp T(a) equilibrates with it: the final temp is between the two original temps, not higher than both.

    Your comment about “not really at equilibrium” just made me see the light (no pun intended): there’s a continuing source of “new” energy that must have a place to go. The radiactive material in the steel gh example, at a temp high enough to keep the planet surface at Xdeg, must be X + Y deg itself. A net flow of energy will continue as long as the surface has a lower energy sink to radiate to. And that sink, the steel shell, will slow down that flow if its thermal conductivity value is lower than that of the surface or the space between. It will “dam up” the egress of energy from the system to the point where the temp of the entire source, surface and shell system are approaching X + Y deg. QED.

    The thermal balance cartoon would make sense if they just change the “324 absorbed by the surface” to “324 maintained in the atmosphere.” Then those Physics Cops would have no case.

    Thanks again for your patience.

  142. Clearly, many of you have not studied Heat Transport. The surface of the Sun is 11,000 degrees F. Anything constantly exposed to the Sun’s rays can reach this temperature if not cooled by something else. Heat only flows from warm things to cool things, not the other way, this is the Second Law of Thermodynamics.
    We’re talking about how CO2 traps heat. Tha analogies to greenhouses are not helpful, much different heat transfer mechanism. CO2 absorbs 15-micron-wavelength IR, and by absorbing it heats the atmosphere, somewhat. Is any 15 micron radiation escaping the Earth’s atmosphere? I have not been able to ascertain this, much BS’ing in the journals. Does anyone know? If there is none, then additional CO2 cannot trap more heat as it is all being trapped now.

  143. Re: Michael Moon (Feb 12 13:01),

    Apparently either you didn’t read my post #63 above or you didn’t click on the link. Look at the spectra here: Observed Atmospheric emission spectra. In the 20 km looking down spectrum (a), there is emission at 667 cm-1 (15 micrometers). However, its brightness temperature is lower because the effective altitude of emission is near the tropopause where it’s much colder than the surface. Rodrigo Caballero has expanded Chapter 5 of his lecture notes so it’s quite a good summary of atmospheric radiative transfer. Section 5.15 starting on page 126 discusses effective emission level, but you’ll need to be familiar with the concepts introduced earlier in the chapter to understand it.

  144. Doesn’t it all really just boil down to the question of what’s the R-factor for our atm with a composition of 20% O2, 79% N2 & 380ppmCO2 vs 450ppm CO2 ? Has anybody measured this in vitro?

  145. Re: guidoLaMoto (Feb 13 06:58),

    Has anybody measured this in vitro?

    That’s not a trivial exercise. For the short path lengths that you would be able to measure in a laboratory, the volume of air would be isothermal and isotropic. The decrease in pressure and temperature with altitude is important and I don’t see how you could duplicate it in the lab in a single measurement. The emission and absorption spectrum of CO2 and how it changes with temperature and pressure has been measured. The data for CO2 and for other molecular gases found in the atmosphere is in the HITRAN database.

  146. DeWitt,
    Thank you for the explanations and the plots. I am curious, do you know of any radiation measurements of the earths surface emissions taken right at, or very close to, the surface (i.e at the surface, looking down)?

  147. Re: kevoka (Feb 13 19:08),

    I don’t know of any actual spectra, but the emissivity of most of the things that make up the surface of the Earth has been measured and, AFAIK, found to be constant and close to 1 for the thermal IR spectral range. So the assumption used in MODTRAN and most other radiative transfer calculations (again AFAIK) is that emission from the surface follows the Planck function at the temperature of the surface with an emissivity of 0.98, that is a gray body that’s almost black.

  148. Wouldn’t a simple insulated tube of air of known composition with a constant heat source at one end and a thermometer at the other suffice to measure the thermal current? Tube would have to be long enough and the heat source cool enough &/or exposure time short enough that the thermometer won’t reach the temp of the heat source. Vary the [co2] and compare. Applying that info to the real atmosphere dynamics may be more complex, but it would at least put us into the right ballpark for estimating the actual effect of [co2] on temps. Gotta be better than a computer program based on some knowledgeable expert’s fantasies.

  149. @181 Kevoka,

    Not sure what you mean by ‘close’ to the surface. There are field portable systems that can measure soil and rock samples spectra in the visible, near infrared and thermal infrared.

    There are also airborne thermal infrared scanner systems, and satellite systems in operation, as well as public and private spectral datasets.

    Typical uses I have seen are underground fire detection, paleodrainage mapping, hydrological surveys etc.

    I would imagine that ‘real close up’ work would get boring very quickly, since it is similar to measuring ambient background noise?

  150. If the relationship of CO2 and temp is logarithmic, what is the base of the logarithm?

    People are claiming that every time CO2 doubles, temperature would go up by X amount (1 degree or whatever). However, if it is a base-10 log function, would it not be 1ppm – 10ppm = 1C, 10ppm – 100 ppm = 1C, 100ppm – 1000ppm = 1C, etc.?

    Or is it a natural log function where increasing the concentration by a factor of 2.7 (approximately) adds 1C to temperature?

  151. All log functions can be referenced to the natural logarithm (it is a simple scale factor), PeterB, so it is really immaterial. In climate science literature, you generally see natural log (ln), but with scale factors. It is just easier to talk about doublings because that is what is expected, i.e., 560 ppm (doubled from the “base” of 280 ppm). Curiously, I’ve also read that there may not be enough carbon in fossil fuels to get much past a single doubling, either.

    Mark

    FYI: for arbitrary base r, log_r(x) = ln(x)/ln(r)

  152. Re: guidoLaMoto (Feb 14 08:14),

    That experiment is basically absorption spectrophotometry. In IR spectrophotometry, the light source is often an electrically heated rod and the detector can be a thermopile. In fact, it was an experiment just like that that led Arrhenius at the end of the 19th century to propose the existence of the atmospheric greenhouse effect and that changing the CO2 concentration would have effect on it.

  153. For lack of a better place, I’ll put this here.

    Which was the greater forcing going from glacial to interglacial temperature regimes, albedo or CO2? Changing the CO2 concentration from 180 to 280 ppmv produces a forcing using the IPCC simplified method (deltaF = 5.3 ln (C/Co) is 2.3 W/m2. Current planetary albedo is about 0.3, so about 103 W/m2 of the incoming solar radiation is reflected rather than absorbed. But the albedo isn’t constant with latitude. First we take the average annual insolation a the top of the atmosphere (TOA) at any given latitude using the 2008 orbital and axial parameters of the Earth, which should do for a first approximation. Then use the amount of solar energy absorbed at different latitudes, for which I use the data from Figure 1.1 in this book to calculate the ratio of incoming to absorbed (1-albedo). The result is this graph. Note the very high albedo at high latitude. So now let’s play.

    If I correct for area and average the current albedo, for the planet as a whole I get 0.31, which is close enough. Using 342 W/m2 as the TOA insolation, the average absorbed is 235.75 W/m2. So suppose we assume that there is no ice and snow at all and that the albedo average for the tropics (-30 to + 30), which equals 0.241, applies to the planet as a whole. Something like this may well have been true at the Eocene peak 55 Mya. That increases the absorption of solar radiation to 259.60 W/m2 for a forcing of 23.85 W/m2 and the planetary brightness temperature from 253.9 to 260.1 K or a difference of 6.2 K. That’s the TOA difference with no feedback. The surface temperature change would be greater. Note that the forcing in the tropics doesn’t change. Any change in temperature in the tropics would be a result of less heat being transferred from the tropics to high latitude because the temperature difference would be a lot smaller than it is now. And that’s also completely neglecting CO2.

    But that’s a really huge change. If I change the albedo so it stops changing at 70 N and 60 S and is a constant 0.4, then the average albedo is 0.293, the forcing is 5.94 W/m2 and the delta T is 1.6 C. So eliminating the permanent polar ice caps is in the same ballpark as doubling CO2 (3.7 W/m2). But that couldn’t happen for thousands of years at least, more like tens or hundreds of thousands.

    Now to see what increase ice coverage would mean. I should do a series, but lets assume that the current albedo from 80 to 90 N and 70 to 90 S remains the same and set the albedo for 50 to 80 N and 50 to 70 S to 0.55, close to the average for 80-90 N. The Canadian border is 49 N for the western part of Canada and we know that ice cover extended at least that far south at the depths of the last glacial period. That gives an average planetary albedo of 0.34, a forcing of -9.92 W/m2 and a delta T of -2.7 K. Again, that’s the TOA, the surface temperature change would be larger. But we’re in the ballpark of the estimated change in the global average. If we add in the forcing from the change in CO2 we get 12.2 W/m2 and the contribution of CO2 is 19%. IPCC WG1 in chapter 6 uses an estimate of forcing from albedo change of -3.2 W/m2. That, IMO, is large underestimate and is the reason that GCM’s require high CO2 sensitivity to produce the required temperature change. The only way I can get an albedo change that small is to restrict the increase in albedo to latitudes greater than 60S and 60N. They talk about vegetation and and other changes, but as near as I can tell, all those would increase, not decrease albedo.

  154. Re: DeWitt Payne (Feb 15 20:18),

    Good thing nobody seems to be reading this because I made a fairly dumb large error that needs to be corrected. The sum of the products is not equal to the product of the sums. Duh. So for albedo = .4 at latitude 60 and higher, the net forcing is 3.2 W/m2 compared to a similar calculation for the current albedo distribution, which turns out to be a global average of 242.7 W/m2 absorbed instead of 235 W/m2. So that fails miserably to explain the much warmer Eocene optimum. The albedo would need to be lower nearer the equator. However, raising the albedo to 0.55 for latitudes 50S to 65S and 50N to 75N still produces a larger forcing than the IPCC estimate, 7.45 W/m2 compared to 3.2 W/m2. That would make CO2 be about 25% of the total forcing. O.55 for the NH may even be an underestimate. A continental ice sheet more than a kilometer thick over the northern part of North America would probably have a higher albedo than sea ice and winter snow. That’s certainly true for Antarctica where the average albedo is about 0.7. That also doesn’t include any contribution for glacial coverage of Chile and Peru in South America.

  155. Arrhenius’ work supported the concept of heat capacity and this was perverted into the greenhouse concept: more co2 (higher heat capacity) should lead to less of a temp increase for any given energy input!

    Re: albedo. Why should land based ice reflect more or less than sea ice? Albedo has to do with reflection, which involves only the surface, not the depth of the ice.

    If increasing snow/ice area via albedo effect has a positive feedback on cooling, and if glaciations did in fact reverse themselves and recur cyclically, then it’s something other than albedo that ultimately drives the cycle. Same argument against co2 as a major player.

Leave a reply to PhilJourdan Cancel reply